Sunteți pe pagina 1din 169

New Questions 2015

Dr Mohammed Talaat, Dr Doaa


abdulhafeez, Dr Duaa Salih Mkawi, Dr
Basel Hisham


Dr Nagwan. Dr You are my choice, Dr
Fiesal hassan, Dr Mona., Dr Mohammed
Alsaadany,Dr Fayez Nofal ,Dr amoona
abdulraheem,Dr al kharaz,Dr Mahmoud
zuheir Dr omar baraa Forgive me if I
forgot some names there is a lot of files.







1

First of All
1. 1-For doing a biopsy of a healthy mucosal gingiva of a 38 years old
patient harming hemidesmosomes is detected by :
A.
lamina dura
B.
lamina luciceda****
2. 2-main characteristic of compound is:
a.
high warpage****
b.
need long time to soften entire mass
c.
water absorbent during kneading
d.
rapid cooling lead internal stress
3. 3-patient with roughness on skin and shiny palms widening of pdl
space but with no ridge restoration and there is a bilatersl destruction
in angle of mandibular bone (he didnt say radiolucency) what is your
diagnosis:
a.
neoplasm
b.
scleroderma****
c.
hyperparathyroidism
d.
aggressive periodontitis
4. 4-The causative in primery apical periodontits:
A.
efeacalis
B.
polymicrobial****
C.
c.gram +ve
5. 5-The function of survey:
A.
To identify the path of insertion***
B.
To identify the rest sat location
C.
To determine the position of major connector
6. 6-Periapical radiograph help in determining :
A.
Pulp health
B.
Canal calcification****
7. 7-Remodelling of the periodontal ligament in orthodontic treatment:
A.
Osteoblast (thats what I remember ) osteoblasts and
osteoclasts
8. 8-In Intrapulpal injection the needle should :
A.
Loosely fit in the pulp chamber
B.
Wedged in the orifice ****
C.
Wedeged in the pulp chamber
9. 9-Chronic renal failure what is developed :
A.
Hyperthyriodism
B.
Hyperparathyroidism****
10.
10-Onlay restoration:
A.
Larger than amalgam restoration****
2

B.
C.

Smaller than amalgam restoration


Same as amalgam
11.
11-Endochondral ossification occure in:
A.
Mandibular body
B.
Maxillary suture
C.
Cranial suture
D. Syncondrosis****
12.
12-Lined by pseuodocollumner with gobeld :
A.
Columner
B.
Gobeld
C.
Sinus****
13.
13-Fracture if the upper complete denture from the labial frenum
to postdam what is the cause :
A.
Ill fitting denture and thick frenum****
B.
Thick postdam
C.
Shallow denture
14.
14-Acute sinusitis is caused by :
A.
Amixed aerobic and anaerobic bacteria
B.
Streptococcus pneumonia****
15.
15-Smile of maxillary incisal edge should be parallel to
A.
Upper lip
B.
Lower lip****
C.
Inferior border of the mandible
D. Mandibular incisor
16.
16-In enamel hypoplasia:
A.
Proliferation
B.
Apposition****
C.
Differentiation
D. In this question I dont remember the rest of the answers
17.
17-In q question about implant there is abone loss around the
implant the answers we
A.
0.5*** it should be from 0.6 to 0.9
B.
b.1
C.
c.2
D. d.4
I have chosen 0.5 and I think it was the right answer
18.
18-Early change result of radiation therapy of oral cavity?
A.
Ischemia
B.
Caries
C.
Mucositis****
D. Xerostomia
19.
19-Difference between K file and reamer?
A.
No spiral fluetes****
3

B.
C.
D.

Shape of cross section


Diameter
Depth of flutes
20.
20-Crown down indicate beginning access?
A.
Small instrument
B.
pre curvature small instrument
C.
remove clinical crown
D. Large instrument first****
21.
22-Length standardized root canal
A.
20
B.
25
C.
16****
D. 30
22.
23-Chemical break composite
A.
Microleakage
B.
Biodegradation****
23.
24-Poly acide modified resin compomer?
A.
No acid based reaction
B.
Filler composite acid***
24.
25- maxillary growth and I cant remember so well?
A.
Interstitial+ intramembranouse
B.
Intramembranouse only***
25.
26. mandibular growth?
endochondral+intramembreanous..
26.
27. a question about a patient taking warfarin and you need to
do surgery.
A.
Pt=1. Ptt=1 INR= 1-1.5***
B.
Pt = 2. Pt=2 INR= 2-2.5
27.
28.Communities with high annual population growth need
education about?
a.
Dental caries. ***
b.
Periodontal disease.
c.
Dentofacial anomalies.
d.
Dental fluorosis.
28.
29-The distal palatal termination of the maxillary complete
denture base is dictated by the:
a.
Tuberosity.
b.
Fovea palatine.
c.
Maxillary tori.
d.
Vibrating line. ***
e.
Posterior palatal seal.

29.
30.We can use for palatal posterior seal:
1. Le jao carver.
2. Kingsley scraper. ***
30.
31-Pt. with lower complete denture, intraoral examination shows
slightly elevated lesion with confirmed border, Pt. history of ill fitting
denture. It is treated by:
a.
Immediate surgical removal.
b.
Instruct Pt. not to use denture for 3 weeks then follow
up. ***
c.
Reassure Pt. and no need for treatment.
31.
32-During 3/4 crown preparation on premolar, bur used to add
retentive grooves is:
A.
Radial fissure.
B.
Tapered fissure. ***
32.
33-Mentally ill child, the best way to apply fluoride:
A.
Acidulated phosphate fluoride.
B.
Natural sodium fluoride.
C.
Fluoride varnish. ***
D. Stannous fluoride.
33.
34-Fluoride reduces caries activity by:
A.
Reduces bacterial adhesion and carbohydrate storage
(antimicrobial activity).
B.
Enhances the precipitation of insoluble fluoroapitite into the
tooth structure.
C.
Fluoride enhances remineralization of the noncavitated
carious lesions.
D. All of the above.
E.
b & c. ***
34.
35-Optimal water fluoridation:
a) 0.5 0.8 mg\liter. ***
b) 0.2-0.5 mg\liter.
C) 2-3 mg\liter
d) 1-5 mg\liter
35.
36-The occlusal reduction for an all metal veneer crown should:
A.
Be as flat as possible to enable an easy fabrication of
occlusion anatomy.
B.
Follow the occlusal morphology with a clearance rating from 1
to 1.5 mm. with the opposing dentition. ***
C.
Follow the occlusal morphology with a clearance of no more
than 0.5 mm with the opposing dentition.
D. Be the last step in the tooth preparation.
36.
37-To provide maximum strength of amalgam restoration the
cavo-surface angles should:
5

A.
B.
C.
D.
E.

Approach 75 with outer surface.


Approach 90 with outer surface.
Be supported by sound dentine.
Be located in area free of occlusal stress.
2+3+4. ***
37.
38-After amalgam triturations, the mix should be placed within :
a.
1 min.
b.
3 min. ***
c.
5 min.
d.
10 min.
38.
39-One of these is less exposed to extensive dental caries:
a- Obes, malnourished
b- Pt. has xerostomia
c- Less plaque score. **
39.
40-Missing lower six and tilted 7:
a- Uprighting of molar by orthodontics. ***
b- Proximal half crown.
c- Telescope crown.
d- Non rigid connector
40.
41-By aging pulp tissue will OR Pulp with age:
a.
Decrease collagen fibers.
b.
Increase cellularity and vascularity.
c.
Decrease pulp chamber size. ***
41.
42.What is the type of sterizliation applied on ligation/fixation
wires: Autoclave. ***
42.
43-Why the moisture heat sterilization (autoclave) is better than
dry heat sterilization (oven) :
A)
Makes the instruments less rusty and blunt
B)
Needs more time and affects the proteins of the cell
membrane
C) Needs less time and affects the proteins of the cell
membrane. ***
43.
44-To enhance strength properties of ceramometal restoration, it
is important to:
a.
Avoid sharp or acute angles in the metal structure.
b.
Build up thick layer of porcelain.
c.
Porcelain should be of uniform thickness and any defect of the
preparation should be compensated by the metal substructure.
d.
Compensate any defect in the preparation equally by porcelain
and metal substructure.
e.
a and b are correct.
f.
a and c are correct. ***
g.
b and d are correct.
6

44.

45-Selection of shade for composite is done: shade guide


a.
Under light
b.
After drying tooth and isolation with rubber dam.
c.
Dry tooth
d.
None of the above**
45.
46-Dentin permeability:
1- Decreases with the increase of cavity preparation.
2- Increase when sclerotic dentin develops under a carious lesion.
3- Increase with smear layer.
4- Decrease in prescence of sclerotic dentin under caries lesion***
46.
47-After class V GI restoration removal of a thin flush of GI is
done by:
a.
Scaller or knife immediately.
b.
Finishing stone immediately.
c.
Scale or knife later.
d.
Finishing stone later.
e.
a + b.
f.
a + d. ***
g.
a+c
h.
d+c
47.
48-Cavity varnish should be applied at least in:
a.
One layer.
b.
Two layers. ***
c.
Three layers.
d.
Four layers.
48.
49-21 years old patient who has iron deficiency anemia, difficulty
in swallowing, with examination of barium sulphate, you found:
A.
Geographical tongue.
B.
Burning mouth syndrome.
C.
Plummer vinson syndrome. *** ( Patterson brown Kelly
syndrome ).
D. Diabetic patient
49.
50-Disinfection of GP is done by:
a.
Autoclave.
b.
Dry heat.
c.
Sodium hypochlorite. **
50.
51-Treatment of internal resorption involves:
a.
Complete exacerbation of the pulp to arrest the resorption
process. ***
b.
Enlarging the canal apical to the resorbed area for better
access.
c.
Utilizing a silver cone and sealer to fill the irregularities in the
resorbed area.
7

d.
e.

Filling the canal and defect with amalgam.


Sealing sodium hypochlorite in the canal to remove the
inflammatory tissue necrotic in the area of the resorption
51.
52-Pt on fixed orthodontic appliance come to clinic after 2
months complaining from beeding with brushing
A ... hyperplastic gingivitis
B .... moderate gingivitis ****
C... deep pocket
52.
53-For post preparation we should leave mm of GP:
a.
2mm
b.
10mm
c.
5mm. *** ( 4 5 mm. )
53.
54-The best restoration for max. central incisor that has received
RCT through conservatively prepared access opening would be:
a.
Post-retained metal-ceramic crown.
b.
Post-retained porcelain jacket crown.
c.
Composite resin. ***
d.
None of the above.
54.
55.Polishing bur have:
a.
Less than 6 blades.
b.
6-7 blades.
c.
10-12 blades.
d.
More than 12 blades. (Carbide bur blades)***
55.
56-Formocresol used in:
a.
Full concentration.
b.
5th concentration.
c.
One fifth concentration. *** 5/1 :
56.
57-Class II composite resin is lined by:
a.
G.I. ***
b.
Reinforced ZOE.
c.
ZOE with epoxy cement.
d.
Cavity varnish.
57.
58-What is the number of pharyngeal "branchial" arches:
a.
4.
b.
5***.
c.
6.
d.
7.
58.
59.Stomodeum and fugi separated by:
1/ buccopharyngeal arch. ( buccopharyngeal membrane )***.
2/ ectodermal cleft
*The stomodeum is separated from the anterior end of the fore-gut by
the buccopharyngeal membrane.
59.
60-AH26 is root canal sealer consists of:
8

a.
ZOE.
b.
Epoxy resin. ***
c.
Steroids
d.
all of the above
60.
61-Scallopped border above inferior alveolar canal between
roots of mandibular molars, this lesion is:
a)
Solitary cyst.
b)
Aneurysmal bone cyst.
c)
Traumatic bone cyst. *** ( = simple bone cyst
= unicame
61.
62-Patient came to your clinic with severe pain, on x-ray the
right side of the mandible has radiolucency with a radiopaque border
that resembles the sunshine rays. Your diagnosis is :
A)
Ossifying fibroma
B)
Osteosarcoma. ***
C) Acute osteomyelitis
62.
63-Pt high mastication and need highly esthetic composite filling
without bevel !***
63.
64-Difference bet symptomatic and asymptomatic apical
periodontitis is
A... number of bacteria***
B... type of bacteria
64.
65.Intrautrine cleft palate at
a 4...6 week
b 8 ...10 week ****
65.
No of pharyngeal arches
5****
6
66.
66-pt has something like injury and bleeding what is the type of
paste u will put it in the tooth to assist heal and the type of local
anathesisia u will use to assist stop bleeding
A.
A.50%bubvicaine+1:200000epinephrine+caoh paste
B.
b.2%lidocaine+1:00000 epinephrine+GI
C.
C.
IN this question no one could remember the answers
67.
67-18 yrs Pt of trauma with central incisor mobile with fracture
of alveolar bone .. what appeared in xray
1 .. break in alveolar bone
2 .. gap bet root apex and alveolar bone***
68.
68-What is the immediate implant
A .. implant that inserted directly after extraction****
B.. restoration of implant
9

69.
69 directlyWhat is patency file ???
A..file that reach apical conistriction
B.. file that with irrigation cause recapitulation***
CONCEPT OF APICAL PATENCY Patency is defined in the American
Association of Endodontics glossary of terms as a canal preparation
technique where the apical portion of the canal is maintained free of
debris by recapitulation with a small file through the apical foramen.
Apical patency refers to the ability to pass a small No. 6-10 K-file
through the apical foramen to assure that the canal is predictably
negotiable. In other words, patent. Buchanan defines a patency file as
a small flexible K-file, which is passively moved through the apical
constriction 0.51mm beyond the minor diameter, without widening it
70.
70-After extraction of lower 6 .. lower 7 tilted to extraction space
.
What is the best ttt
A.. orthodontic ..
71.
71 Flap used to fix bony un attached pocket to be attached
again
1. Apical repositioned flap***
2. Coronal repositioned
3. Modified widman
4. Not displaced flap
72.
72. Nine years old pt with missing mandibular primary canines
bilaterally with otherwise normal dentition for his age and low caries
incidence
What is the most probable cause of missing teeth
1. Previous extraction
2. Congenitally missing****
3. Traumatic avulsion
73.
73. Bone assesment of abutment teeth
1. Horizontal bone loss is more important****
2. Vertical is more imp.
3. Equally imp.
4. Not imp.
74.
74.Avulsed tooth in
a.Saliva
b.Water
c.Blood
d.Low fat milk***
75.
75.Provisional restoration in anterior ceramic preparation
a.GI CEMENT
b.POLYCARBONATE***
10

c.AMALGAM
d.COMPOSITE
76.
76.Swelling in submand gland and discomfort during eating
a.Dentigrous cyst
b.Sialothiasis ****
77.
77.C shape canal found in:
a.Upper 6
b.Lower 6
c.Upper 7
d.Lower 7****
78.
78.In deep cavity we do
a.Caoh gi cement varnish****
b.Caoh varnish gi cement
79.
79.Moist heat better than dry heat
a.Less dull instrument
b.Kill all cell microorganisms**
80.
80.Implant completely made from? Titanuim***
81.
During extraction root or tooth go to max sinus
Left and pt re informed
Remove after 3 months
Remove as soon as possible****
.83
Lesion In tubrosity.82
Flap incision
Blade 15 to remove
Using bur to remove

.82

Red nodule solitory in maxillary lateral incisor area .. soft and .82
? febrile when touch.. pregnant
***Payogenic Granuloma
Hematoma

.83

.a

.b

?Blue black shade in gingiva margin. Patient with gastric reflex .83
**A. Bismuth
.B. Arsenic
Sabecous gland in the scalp...A. Gardner syndrome .84
11

.85

.84

Patient under prednisone 10 mg for 2 months. To prevent adrenal .85


.crisis
Double the dose in the same treatment day**** = double the
dose to prevent adrenal crisis
Mechanical tooth brush invented in 1939 .86

.a

.87

.Bite wing x ray for all except: peri apical lesion .87
?Peri apical x ray: need to be compared .88

.86

.88

.89

90.
89. When you prepare class I I cavity... you found you do not
have gingival seat.?
1. Built gingival seat GI***
2. Amalgam gingival seat
3. Extend to root
91.
90.Question in ethics but was in hard English words that I could
not understand the meaning so I answered all of above
92.
91.Patient come complaining from bad taste and bad smell from
newly fixed three unite FPD .. when exam it ... bubbles are coming
from one of the abutment under applying pressure on it with water
sepray?
a. 1.Fracture joint
b. 2. Open margin
c. 3. Lose abutment**** Separation of retainer from the abutment
93.
92.Mandibular growth occur?
1.After maxilla growth start
2. Together with maxilla
3. Just before maxilla ****
4. Long time before maxilla
94.
93.Mandible from which pharangyal arch?
1. first arch****
2.second arch
95.
94.Paget's disease of bone all correct except?
1. Bone pain
2. Pathological fracture
3. Delay healing of fracture****
4. Hypercementosis
5. Osteoarthritis
96.
95. 1 years old patient diagnosed with herpes simplex acyclovir
given to him what you should give to him?
1. Protein with pain killer
12

2. Topical Aneasthetic only


3. Multivitamin with topical aneasthetic*****
97.
96. Occlusal Force on RED IS distributed by?
1. Proper location of rest
2. Double Ligual bar major connector
3. Wide occlusal table
4. All of above****
98.
97.After surgical doing the crown lengthing..why we have to wait
for 6 month?
a. complete cemantosis
b. complete epthilum ***
99.
98.After the RCT done...where the spreader should be pass in
obturation?
a. to working length
b. shortar than radiographic apex by 1 mm...****
c. to th radiographic apex..
100.100.When the dentist do pulp cappping?
A-reversable pulpits***
B-irreversable pulpits
101.101.In deep cavity hard caries in dentin...who U.tretment?
a. A-low speed
b. B- carbide bur with high speed ****
c. c- excavator...?
102. 102.What is the material can be used as a base to composite?
a. ZOE.
b. GIC.
c. caoh.
d. D-zinc phosphate.
e. ALL
f. - a+b
g. The correct answer is C and B****
under the amalgam restoration?321 .103

.103

.Varnish .1
Gic.2
.Ca (OH)2 .3
****321 amalgam
104.104. Pt. wears complete denture for 10 years & now he has cancer
in the floor of the mouth. What is the first question that the dentist
should ask?
a- does your denture is ill-fitted***
13

b- smoking. (80 % of the cancer of the floor of the mouth is caused


by smoking ).
c- Alcohol.
d- Does your denture impinge the o. mucosa. (traumatic cause).
105.105. A patient that wasnt anaesthetized well in his 1st visit, next
day he returns with a limited mouth opening (trismus) He must be
anaesthized, whats the technique to be used:
a) Williams technique.
b) vazirani-akinosi technique *** (Berchers technique)

106.106.To remove a broken periodontal instrument from the gingival
sulcus:
a) Schwartz Periotriever. ***
107.107.An adult had an accident,, pulp
complex fracture coronnaire
incomplex
luxation. ***
subluxation.
laceration.
abrasion.
contusion.
108.108.Treacher collins syndrome characterized by :
A-Prognatheic of mandible.
B-No hearing loss.
C-Upward sluing of eye.
D-Malar Bone NOT well formed or absent. ***
* Malar bone = Zygomatic bone = Cheek bone.
* Treacher Collins syndrome = mandibulofacial dysostosis.
face fish like due to underdeveloped cheek bone, down sluing of eye
retrognathic of mandible, fissure palatal, open bite, hearing loss
109.109.Blood supply of floor of the mouth? Lingual artery****
110.110.Perfoeation in TMJ
MRI
Cephalometric
Arthrogaphy *******
111.111.smoker 60 cancer 40 control non smoker 10 90
cancer Odd ?
Odds ratio = 90*60/10*40= 5400/400 =13.5.****
N.B The same table is written in a question in a better way describing
the answer and the question clearly ( question number 616)
112.112.Bacteria present in pericoronitis?
Strepto. Malleri or Bacteriod, Streptococcus , spirochetes
14

113.113.Best X-ray used to diagnose fracture of the angle of the


mandible?
a. antero posterior
b. lateral oblique 30 degree angel***
114.114.Kidney function assessment tests include ?
a. BUN,Creatinine
b. Bun, Creatinine,K,Ca****
c. K,Ca,Cbc
115.115.amalgam restoration fracture caused by?
a. Improper cavity ***
b. Contamination
c. condensation
116.116.What is the syndrome associated with hand and foot
keratosis,Periodontitis affecting both dentitions,early teeth loss and
generalized bone destruction.?
Papilon le Fever***PLS
117.117.For Nasal fracture Treatment ? Walshams forceps.***
N.B if nasal bone is deviated it will be straightened by Achs forceps
afterwards.

118.118.The periodontal ligament compromise which of the following?


a. Gingiva and the Pdl
b. Gingiva , Pdl and The alveolar bone
c. Gingiva,Pdl,alveolar bone and Cementum****
15

d. Gingiva,Pdl,alveolar bone,cementum and enamel


119.119. When a child must first exposed to the use of the tooth brush?
a. Immediatly after eruption of first tooth****
b. age of two years
c. age of 4 years
d. at the age of primery school
120.120. Fluoridated tooth paste for least 3 years old child is ?
a. recommended****
b. limited
c. Toxic
121.121.Repeat fluoride topically in clinic does not cause fluorosis
because ?
a. a.It is not the same fluoride that cause fluorosis
b. b.Dentist removes excessive amount
c. c. teeth already calcified ****
d. Saliva was not out
122.122.Post length increasing will ?
a. increase retention****
b. increase resistance
c. increase strength of restoration
d. d .decrease retention
123.123. Polishing Bur have ?
a. a.Less than blades
b. b. 6-7 blades
c. c. 10-12 blades
d. d.more than 12****
124.124. Moisture heat sterilization over than dry heat sterilization ?
a. a.makes the instruments less rusty and blunt
b. b. needs more time and affects the protiens of the cell
membrane
c. c.needs less time and affects the protiens of the cell
membrane****
125.125. Bacteria in root canal infection?
a. mixed anaerobe and aerobe****
b. b.single obligate anaerobic only
c. c.aerobic only
d. d. non of the above
126.126.Autoclave relative to 100F dry oven?
a. a.the same time
b. b. less time ****
c. c.slightly higher time
d. d.considered higher time
127.127.Facial nerve supply?
16

a. masseter muscle
b. temporal muscle
c. buccinators muscle****
d. mylohoid muscle
128.128. One ampule of L.A in 2% Lidocane with epinephrine 1/100000?
a. a.0.001 mg
b. b.18 mg
c. c. 0.002mg
d. d. 36mg****
129.129. AU- shaped radiopaque structure in upper first molar x-ray is ?
a. a.Zygomatic
b. b.Maxillary sinus wall
c. c.Zygomatic process****
130.130.When IND block for child should be ?
a. mandibular foramen above occlusal plane
b. b.mandibular foramen bellow occlusal plane****
c. c.mandibular foramen lateral occlusal plane
d. d.mandibular foramen anterior to occlusal plane
131.131.The body secretes antibody against antigen using which cells?
1. T-lymphocyte
2. B-lymphocyte***
3. Plasma
132.132 Principles of elevator uses all of the following except ?
a. a.Wedging the socket wall****
b. b.Wedging
c. c.Wheel and axel
d. d. Lever
133.133. Photograph for perforation in disc of the Tmj?
a. MRI
b. b.Ct
c. c. Arthrography****
d. d. Lateral cephalometric
134.134.Gastric secretion decrease by?
a. H1
b. b.H2
c. c. Anticholenergic****
d. d. Cholenergic
135.135. Drug used to decrease saliva during impression taking ?
a. A.Cholinergic
b. b.Anticolinergic****
c. c. Antidiabetic
d. d. Anticorticosteroid
136.136.Patient on long term antibiotic came with systemic candida ?
17

a. a.Amphotracin
b. b.Fluconazole****
c. c.Penicillin
d. d. Antihytamine
137.137. After bleaching tooth we want to restore the tooth with
composite resin we do not want to compromise the bonding we wait
for ?
a. a.24 hours
b. b.a week****
c. c.One hour
d. d. Choose different material
138.138. CMCP contains Phenol in concentration
a. 65% Phenol , 35% compher
b. 35% Phenol, 65% compher****
139.139.65 years old black man wants to have very white teeth in his
new denture what should the dentist do?
a. a.Put the white teeth
b. b.Show the patient the suitable color first then show him the
white one
c. c. convince him by showing him other patients photos ****
d. d. tell him firmly that his teeth color are good
140.140. during clinical examination the patient had pain when putting
probe gently on the root what is the diagnosis?
a. a.Dentine hypersensitivity****
b. b.reversible pulpitits
c. c.irreversible pulpitis
d. d.apical periodontitis
141.141. The most superior way to test the vitality of the tooth with ?
a. Ice pack
b. Chlor Ethol
c. Endo special ice ****
d. Cold water spray
142.142.Avulsed tooth is washed with tap water it should be replaced
again ?
a. A, Immediatley****
b. b.After 4 hours
c. c. 4days
143.143. Patient have a complete denture came to the clinic tell you no
complaint in the talking or in chewing but when you examine him you
see the upper lip like too long deficient in the margins of the lip?
a. a.deficiency in the verticle dimension
b. b.male position of anterior teeth ****
c. c. deficient in Vit B
18

144.144.Patient that has the central incisor with sever resorption and
whos going to make him extraction of the premolar which of the
following is not correct procedure of treatment
a. A.Rpd
b. b. Implant
c. c. Maryland bridge
d. d. autoimplant of the premolars****
145.145.Patient under corticosteroids therapy he will undergo surgical
extraction of the third molar what will you give him to avoid adrenal
crisis ?
a. a.Dixamethasone(4mg/IV)
b. b.Methyl prednisone (40-60mgIV)
c. c. Hydrocortisone sodium sulfide(40-50mg)
d. d.Hydrocortison sodium succinate (100-200mg)IV or IM****
146.146. Scale to measure marginal deterioration ?
a. Mahler scale****
b. b.color analogues scale
147.147.Proxy brush with which type of furcation involvement?
a. furcation Grade 1
b. Furcation Grade 2
c. c. Furcation Grade 3****
d. D.Furcation Grade 4
148.148. Generalised lymphadenopathy is seen in?
a. infection
b. lymphocytic leukemia
c. HIV
d. d.perniciouse anemia
1. a+b
2. a+b+c ****
3. Only d
4. B+D
149.149.when removing most caries dentin which exposes the pulp
.dentist should ?
a. a.do direct pulp capping
b. b. do indirect pulp capping
c. c. prepare for endodontic treatment.****
150.150. child with dental caries in 3 or 4 surfaces if his first primery
molar we will replace them with?
a. performed metal crown****
b. porcelain crown
c. amalgam
d. composite restoration
19

151.151. Patient with pain on the upper right area and the patient can
not tell the tooth causes the pain what is the least reliable way to do
pulp test ?
a. a.cold test
b. b. hot test
c. c.electric test****
d. d.stimulation of the dentin
152.152.Periodontaly involved root surface must be root planed to?
a. remove the attached plaque and calculus
b. b.remove necrotic cementum
c. c. change the root surface to become biocompatible
d. d. Aand B only****
153.153.all these are right ways to handle instrument except ?
a. a.modified pen handle
b. b. inverted pen
c. c.pen grasp****
d. d.palm and thumb
154.154. Die ditching is ?
a. a.carving apical to finish line ****
b. b.carving coronal to finish line
c. c.mark finish line with red pen
155.155. When you want to make immediate complete denture after
extraction of all teeth what type of suture you will see ?
a. a.Horizontal mattress suture
b. b.verticle mattress suture
c. c.interrupted suture ****
d. d.figure 8 suture
156.156.you sent a shade to PFM technician gives you different color
with same shade ?
a. Thick opaque
b. non uniform porcelain****
c. thin opaque.

20

Operative and endo


can u swollow mercury every day - 157
A/10mg/day
***B/5mg/day
C/60mg/day

.157

Child came to clinic with history of injury trauma from one year - 158
and rct..then central incisor dark colour..the cause is
*** A) not complete remove of pulp
B) bleeding from pulp
C) contamination gutta perch

.158

Calcium hydroxide provide - 159 .159


*** A) can enhance form hard tissue in root
B) seal canal
:AH26 is a root canal sealer -160
.A- contains zinc oxide
.B- contains steroids
*** .C- is an epoxy resin
D- all of the above

.160

For the prevention of mercury toxicity - 161 .161


:in the clinic put it in
.Water .1
.Sodium chloride .2
.The appearance of radial .3
Fixer of radial. *** or container that well not porn .4
162.161 - The only advantage of plain gi without additives over gi with
additives:
1- streghth
2- less contraction ***
3- shelf life
4- rapid setting
163.162 - The exact describtion of healing in endo treatment with
radiolucency related to apex and fistula:
1: disappear of radiolucency
2: asymptomstic tooth
21

3: decrase in radiolucncy
4: disappear of fistula ***
164.163 - Most retentive crown is :
a. 1 Full metal ***
b. 2.Veneer
The root of upper lateral incisor inorder to be best radiographed - 164
use
***A.paralling technique
B.bisecting technique
C.panorama
Simpifil type for canal enlargement by NiTi in: or best NITI - 165
rotary file is
***a. Universal protaper
B. Reciprocal
c. Revers S
Base under composite posterior restoration 166
A zinc poly carboxilate****

B ca(oh)2

.167

.a

After trituration of amalgam condensation must be - 167

.168

*** A.after (3_4)min.at least in order to remove excess mercury


B.vertically
C.with little pressure
:In root of internal resorption the pulp is - 168

.169

A.Reversible pulpitis
****B.Irreversible pulpitis
C.necrosis
:In root of external resorption the pulp is - 169

.170

A.Reversible pulpitis
B.Irreversible pulpitis
**** C.necrosis
?Which pathogen(s) in primary apical periodontitis - 170
A- pnemonas fast lies
22

.171

.166

.165

*** B- many microbial spa


C- aerobic
Ortho and pedo
teeth which responsible for crowding teeth 171
A/1ry lower first molar
*** B/1ry lower second molar
C/permenant first molar
D/1ry max secon molar

.172

173.172 10 years child presented with space between maxillary incisors


and have problem in overjet .treatment????
1.fixed appliance
2.removable appliance
3.normal no ttt *** (ugly duckling stage )
Pathfinder survey 173

.174

A-At 5yr in primary teeth ,15+25+25+60.in permanent teeth


B-5-10 in primary
C-5 yr in primary
D-5-10in primary 30,45,55,65 in permanent
*** 5,12,15,35,44,65,14
175.174 - 8years child had trauma in upper central incisor which reveal
pulp exposure and the pt under go with caohpulpotomy the way to
distingushsucess of ttt is
a . Asymptomatic tooth
b .Respond to electric pulp test
c .Continue dentin formation and apical closure ***
176.175 - Pt have cross bite when move his mandible laterally?why
1.unilateral costriction of maxillary
2.unilateral constriction of mandible
3. Asymmetrical growth of mandible
4.bilateral constriction of maxilla****
177.176- Hand mouth technique .use
1 - punishment
23

2- -ve reinforcement***
3-*ve reinforcement
178.177 - What is the most important teeth to prevent the severity of
crowding:
a. Upper E
b. Upper D lower E ***
c. Lower d
179.178 - age of most traumatic injury to primary teeth.
2:3 ***
Less 2
5::6
7:8
180.179 - trauma to primary teeth mostly cause
1 intrusion ***>60%
2-root fracture
3- tooth fracture
4-bone fracture
181.180 - Nomal range in gingival depth ( epithelial attatched ) in health
mouth
A 1 to 2
B 2 to 3 ****
C 0 to 3
D 0 to 5
A healthy sulcular depth is 3 millimeters or less.However, in certain
situations, a gingivectomy is necessary to reduce the gingival pocket
depths to a healthy 13 mm.
182.181 - Early extraction affect :
A.speech and Occlusion***
B.speech
C.appearance
D.appearance and speech
183.182 - Lost mandible 6 and the 7 is tilted best treatment
a. A ortho***
b. B proximal half crone
c. C microscope crown
d. D nonreged connector
184.183 - 12 year patent lost 63 and 64 what the treatment
a. Intrem RPd
b. Leave ****
185.184 - Posterior bit plate is used to :
a. Crowing posterior
b. Anterior deep bite and distal movment of ant .teeth ***
24

Prosthodontics
186.185 - There's pain in 44,34 whih it apartment of RPD what's the
reason
A.there's no stress bleaker ***
B.problem in major connector
C.problem in supporting structure ***
187.186 - Best material for impression of flabby tissue:
1- plaster of paris ***
2- agar agar
3- silicate
4- zinc oxide
188.187 - dentist at the end of the day want to pour alginate imp quikly
how can he do that
a.increase powder/water ratio
b.hot water ***
c.slurry water
d.increase thickness ...............
189.188 - Co cr RPD. Occulosal rest here to
1- retention
2- stability
3- reciprocation
4 - strength of design
5- support ***

Apicture of case - 189 .190


Lower Class I PD with 4 anterior teeth with recession grade ||,what's
the ttt
A.extraction of all teeth and make single denture
* **B.over denture after teeth preparation
C.Gum stripper
How can I make appearance of abutment smaller without - 190
change in dimensions
A.make mesial and distal line angles near teach other
25

.a

.191

*** ake cervical hight of contour more incisally B.m

.b

Child requires graft in his alveolus what is the best graft - 191
****a. Autogenouscancellous
b. Freeze dried
c. Autogenouscorticocancellous

.192

193.192 - After perio surgery we wait 5 ::6 month in order to


1 complete re epithelial
2- complete maturation***
194.194- Patent with stone in submadiblar gland and have allergy to
iodine what type of scanning occlusal film
Surgery
Lefotr 1 injury- 195 .195
A/greater platine artery
B/infra orbital artery
**** C/maxillary artery
D/mandibukar vein
use irrigation when cutting bone due to - 195 .196
A/prevent risk of infection
B/remove bad smell of bone
* C/heat generation during cutting bone affect on bone vitality
The local anesthesia depend on - 196 .197
A)strength bond between drug and nerve
* B) strength bond between drug and its intensity
C) bond between drug and time of removal from body
Paranasal fluid occure in the fracture of the face - 197
A. Lifort I
B. Lifort II
*** C. Lifort III
D. Zygomatic fracture
E. All above

26

.198

:Salivary gland disease ( tumor ) with perineural invasion - 198


.Pleomorphic adenoma .1
*** .Adenocyctic carcinoma .2

.199

200.199 -Patient will have multiple extraction what to do after extraction


for denture sake:
1- make intrupted suture cross papillae
2- leave to heal to avoid elevations from sutured papillae***
3- make extensive bone smothning
4- put surgical pack only
201.200 -the origin of innervation of post thid of tongue
a.trigeminal
b.lingual
c.hypoglossal
d.glossopharyngeal **
? Best biopsy type- 201
Incisional

.202

.a

*** Excisional

.b

?Which artery supply floor of the mouth - 202


*** lingual artery.1
inferior alveolar artery.2

.203

204.203 -Mandibular growth at chondyle


1 interstitial&chondro replacement
2 appostion&chondro replacement ****
3 appostion& intra membranous
205.204 -Hyper cementosis character
1 - difficult in extraction
2- in paget disease
3 - bulbous root
4- very successful extraction by elevator
1&2&3 ****
:Pt. with chronic renal failure what is developed - 205
A. Hyperthyroidism
***B. Hyperparathyroidism

.206

.Asking about disinfectant of dental chair after HBV - 206


****iodophore&hypo clorid -1
27

.207

formaldehyde -2
ethylene oxide gas -3
ethyl alcohol/detox /100 -4
2&1
3&2
4&3
Mechanism of mandible growth atcondyle- 207
Apposition and intramembranous modeling
Interstitial and endochondral
*** Apposition and endochondral
Interstitial and intra membranous
:Lingual nerve branch of- 208
a. Trigeminal N
***b. Mandible N
c. Facial N

.208

.209

:Lingual nerve- 209 .210


A. part of trigeminal nerve
B. supply the submandibular gland
*** c. A and B
211.210 -Pt need exo, he takes Antidepressant, amount of epinephrine
on anesthesia:
a. 0.1
b. 0.02***
c. 0.4
d. 0.8
Child 6 year have abnormal enamel dentin and pulp in A - 211
:quadrant you diagnosis is
a. hypoplasia
*** B. regional odontplasia
c. Detogensisimperficta
d. Amelogensisimperficta
213.212 - Which pathogen(s) in primary apical periodontitis?
A- pnemonas fast lies
B- many microbial spa ***
C- aerobic
28

.212

214.213-What type of L.A have the slowest onset.


1-procaine ***
2- lidocaine
3-bubificane
215.214 - What is formed from epithelial rest of serres
A.epithelial root sheeth of hertwing
B.laminadura
C.vestibular lamina *
N.B root formation, and once complete, it degenerates, leaving
epithelial rests of Malassez
Residual dental lamina, from bud-like invagination process of tooth
formation, is source of epithelial rests of Serres, located primarily in
gingival soft tissue
216.215 -Which cranial nerve when injury responsible for gag reflex.
1- V
2-7
3-9 ***glossopharangial
4-sinal branch of 11
217.216 - Pharyngeal arch called.
a. A maxillary arch.
b. mand arch. ****
c. C thyroid.
d. D hyoid***
e. B and D.are the right answers if the question is not wrong***
218.217 -TMG &disc direction displaced
Anterior***
Posterior
Lateral
?What different between center of the growth and site of growth .218
**** independent
centered

.a

.b

**the center of growth is rapid

.c

Mandibular growth at chondyle- 219 .220


interstitial&chondro replacement 1
*** appostion&chondro replacement 2
appostion& intra membranous 3
29

.219

221.220 -In order to activation of periodontal instruments the blade


should make angle with facial surface of the tooth
1- 45:90 ***
2- 90:180
3- 15:30
4-30:45
222.221 -Question about hornar syndrome
223.222 - Bone between 2 root parallel to :
A gingival****b pdl
224.223 - of the lower lip origen from :
a. A - Mandible branch of pharngial arch****
b. B lingual branch of pharngial arch
225.224 - Child 3 year old 15 kg maximum carpole (local anesthesia )
can you give him
1
2 ****
3
4
5
226.225 - Low painful anesthesia by :
a. Needle gage more than 25
b. Stretch the tissue
c. Use topical
d. All
e. B, C best answers
Bacteria
bacteria which not found in peridonditis - 226 .227
*** A) streptococcus
B/staphylococcus
C/bacteroids
D/famulis
Bacteria in - 227
?pericoronitis
Staphylococcus
**bacteroids
famulis

.228

.a

.b

.c

229.228- 28 years old pt reporting to aspecialized clinic ------------periodontitis ,after phase 1 of the treatment plane the pt is advisefore
full mouth periodontal flap surgery after surgical debridment creater
30

are present in intel septal bone. This osseos defect are best
eliminated by?
a. Osteoctomy***
b. Osteoblasting gingivectomy
c. Subgingival curettage
d. ------------ osteoblast
230.229-Best treatment for furcation perforation Mineral trioxide
aggregate
231.230-antibiotics prescription is recommended when:
A-diffuse rapid spreading infection***
B-acute localized infection
C-chronic infection
232.231-Antibiotic prescription is recommended in
a. Pt with cellulities and enlareged lymph node****
b. Pt with acute inflammation with swelling
233.232-definition of 3ry dentin:
a. a.Irregular 2ry dentin
b. .Scelerotic or
c. reparative
d. a,c***
e.
234.233-pt has hemophelia and wantto extract lower carious molar what
is the additional techq give him if he still fealing pain
a. a.Inferior alveolar nerve block
b. b.intraosseous ingection
c. c.supraperostial ingection
d. d Intraligamental****
235.234. pt has something like injury and bleeding what is the type
ofpaste u will put it in the tooth to assist heal and the type of local
anathesisia u will use to assist stop bleeding
a. A.50%bubvicaine+1:200000epinephrine+caoh paste***
b. b.2%lidocaine+1:00000 epinephrine+GI
236.235. -Child suffered from wheezing , shortness of breath, severe
coughning during dental treatment the cause is:
A-Asthmatic attack***
B- cardiac attack
C-adrenal crisis
D-insulin shock
237.236.TTT FOR juvenile periodontites
a. a.tetracyclin****
b. b.calyndamycin
c. c.penecillin
31

Both A and B are true A is better cause always in refs its written first
the second thing is that I have read that its very powerfull agains AA
unlike clyndamicine AA is the main causative micro organism for
Localised aggressive periodontits.
238.237.we can use to creat palatal posterior seal
a. 1.LE CRON CARVER
b. 2.kingly scalper****
239.238.Cement that if mixed slowly give good working time
a. Znph****
b. Caoh
240.238.pt 28 come no complain make rotein periapical x ray u found
radiolucent lesion related to lower incisors no pain no cavity no
swilling the tooth is sound
a. Periapical granuloma
b. Ossifying fibroma ****
c. Radicular cyst
systemic antifungal 239

.241

***a fluconasole
b amphotericin b

consious sedative for children .240


a Naloxone
b Flumazenil
****c Benzodiazepines
d promethazone

.242

lesion 2 cm of oral cancer, no metastasis, no spread to regional .241


lymph nodes
And asked for the right TNM classification
T0N0M0-1
T1N1M0-2
3-T2N1M0
4-T2N2M1
The right answer is T1N0M0
244.242.To determine marginal leakage: maheler scale
245.243.for rotated teeth after ortho we should do
32

.243

a.circumferential fiberotomy****
b.circumferential frenictomy
246.244.pt come after one day from doing amalgam restoration to
upper 7 and appose lower 7 with gold restoration complain from
galvanic pain what u will do regarding that the amalgam restoration
with good anatomy and occlusion
a.Change restoration
b.Put varnish****
247.245.child 8 yrs old come with miniute exposure with open apex
upper central incisor
a.Pulp capping****
b.Pulptomy
c.Apexfication
248.246.what is the space by millimeter should leave for cement under
ceramometallic crown
a.2-5mm***
b.5-10mm
c.10-30
d.30-60
249.247.frist formation of tooth bud intra utrine life
a.4
b.5
c.6week
d.8****
N.B Bud stage starts at 8 weeks intra uterine while .First sign of tooth
formation is at 6th week intra uterine. And first sign of calcification Or
appositional stage is at 14th week intra uterine.
250.248.pt come for construction of CD with enlarged mandibule and
maxilla and pulged forehead the x ray show radiopacities and
hypercementosis and cooton whool appearance with high level of
potassium sulphatePagets disease of bone***
251.249.10yrs old pt with painless bilateral swelling of mandibule wich
is firm multible RL
Cherbuism***
252.250.child 14 month come with his parents ulcers or visecles on mid
palatine raphe
Epistien pearls***
253.251.female pt with dry mouth dry eye high caries index Sjogrens
syndrome***
254.252.deformty in malar bone.treacher Collins
255.253.cleft lip and palate with hypodontia..vander waund
syndrome
256.254, the best impression for pt wth onlay .polyvenyl siliconase
33

257.255.cu added to amalgam to decrease gumma phase13%


258.256.diseased root u want to remove it to assist remaining parts of
multirooted teeth to survive
a. Root amputation
b. Root resection****
259.257.65 yrs old.pt come for construction of CD with history of 2
complete dentures in last one year the pt ask about details of every
step and want to know more details
a. Philosophic pt
b. Hystrical pt****
260.258.pericronities bactria
Peptostereptococcus+strept.millere+fusiebacteria
261.259. - The concentration of household bleach is:
A-5.2%***
B-2%
C-8%
D-0.5%
262.260.width of GP tip 30 after remove 1mm0.32
263..261.grade 2 furcation and grade 2 mobility tooth with short root
healing will be
a. Fair****
b. Moderate
c. Poor
264.262.GTR removed after 4-6
265.263.carious tooth with radiopaque lesion
a. condensing osstities****
b. cementoblastoma
266.264.female pt with CD WITH angular chelities and wrinkles around
her mouth.
a. Low vertical dimention***
267.265.pt has CD with low vertical dimention and wrinkles around her
mouth she suffer from
a. a.angular stomatitis***
268.266.pt want to construct RPD with remaning 17 with good
restoration and no mobility and missing 26 27 28 what is the type of
clasp u will choose to 17
a. a.aker clasp
b. b.ring clasp
269.266. the besT cement give esthetic under partial veneer restoration
a. a.Light cure resien****
b. b.Reinforced resin cement
270.267.the type of crown for fractured upper central incisor
34

a. a.Porcelain fused to metal****


b. b.Ceramometallic restoration
c. c.All ceramic crown
271.268..x ray for upper lateral
a. a.parallel techq***
b. b.bisecting angle techq
272.269.the function or the role of adrenalin in endosurgery
a. a.prolong duration of anathesia
b. b.bind with alpha recebtors in
c. c.bind with beta recebtors in.
sorry I cannot remember but the option of vasoconsterctor
wasnnot present
heamostasis**** is The Right Answer.
273.270.containdication of ginevectomy
a. a.suprabony pdl abcess****
b. b.gingival hyperplesia
c. c.type of surgery but cannot remember
274.271.the favorable position for margin of crown end at
a. a.on gingival crest
b. b.above gingival crest****
c. c. below
275.272.ttt for mucoscele..excesion****
276.273.child pt come with his parents with blue swelling on the gingival
with excessive salivationEruption hematoma***
277.274 . extrusion of sealer from canal into periapical region
a.lack of apical stop****
b.small con of GP
278.275.caries detection die.propaylen glycol
279.276.full fear child girle give her
a. Promethazone
b. Benzodiazeben****
280.277.which best time for pt want to make composite after bleaching
a. At the same day
b. After week
c. After 2 week****
281.278.preparation of groove for partial veneer restoration By tapered
fissure
282.278.width of artificial teeth for CD should
a. Less than natural teeth****
b. Equal natural teeth width
c. Larger than natural teeth
283.279.retraction of flab and cheek togetherMinnesota retractor****
35

284.280.pt want to RPD and has inoperable torous palatinus the type of
major connector
a. Horse show shape****
b. Ant palatal bar
c. Ant post palatal strap
d. Post palatal strab
285.281.best pulp capping material for children
a. Caoh***
b. Zno
c. Formcresol
286.282.child come to clinic with amalgam fracture in ismuth portion
due to
a.High occlusion
b.Wide preparation at ismuth portion****
there arenot option for shallow preparation
287.283.picture for white lesion on mucosa of cheek and ask about pt
with renal transplantation and has white lesion which is
a. Hairy leukoplakia****
b. Idiopathic leukoplakia
288.285.child pt with history of hepatitis c, history of stress Lichan
planus***
289.286-Which instrunment we use to restore maxillary bone if its not
fullt detached in lefort one ? Rows disimpaction Forceps***
290.287-Which type of cells affected in diabetic patient which increase
the ability of infection?
291.289-Whats the injection technique which used as a supplement to
the anesthesia for the extraction of lower molar ?
292.290-Patient came to dental clinic after amalgam filling in the same
day to check the electric shock which he has after examination
dentist noticed that there is another metal crown on the same area
upper and the filling is not stable what should he do ?
a. Separating medium
b. Change the filling ****
c. Varnish
293.291-Characteristics of verrucus vulgaris?
294.292-How is the internal resorption appears on x-ray?
295.293-Whats the affect of saliva on the filling materials
a. Softens resin
b. Change the filling shade ****
c. Increase the bond between the tooth and the filling
36

296.294-Whats the duration after obturating root canal exposed to oral


cavity ?
a. 2 weeks
b. 6 days ****
c. 3 months
d. 6 months
297.295. What is the best material for direct pulp capping in pedo:
A. zinc oxide
B. formocresol
C. caoh****
d. GIC
298.296. 10 years child presented with space between maxillary
incisors.treatment?
1.fixed appliance
2.removable appliance
3.no ttt****
299.297. We want to construct upper denture with palatal strap. Which
act as indirect retainer... what type of Kennedy class:
A. cl 1
B.cl2
C. cl 3 ****
D.cl 4
300.298. 10 years child with congenital heart disease came for
extraction of his lower 1st molar, the antibiotic of choice is:
a-Ampicillin 30 mg /kg orally 1hour before procedure.
b-cephalexin 50mg/kg orally 1hour before procedure.
c-clindamycin 40mg/kg a-Ampicillin orally 1hour before procedure.
d-Amoxicillin 50mg/kg orally 1hour before procedure.****
301.299. Zinc polycarboxylate cementis better than zinc phosphate
cement in:
a- Compressive strength.
b- Low solubility.
c- Film thickness.
d- Adhesion to enamel.****
302.300. - Primary teeth have:
A.Normal pulp canals
B. straight bulp canals
C. tortuous pulp canals****
D. regular pulp canals
:Adjacent to PDL which bone the fiber inserted to it .301
****A. bundle bone
37

.303

B. alveolar bone
C. cancellous bone
304.302. Patient presented to you after fitting the immediate denture 5
10 months, complaining pain and over tissue in the mandibular,
what is the diagnosis:
1. Epulis fissuratum. ****
2. Hypertrophic frenum
305.303. Pt. with complete denture 2 yrs. ago, come with small red
lesion, he has bad oral hygiene and wear denture all time at night
also diagnosis is:
A. denture stomatitis
B. epulis fissuratum****
306.304. - diabetic patient and allergic to penicillin with swelling dueto
caries :
1-give clindamycine then RCT
2-RCT and erythromycin ****
3- RCT and antibiotic
307.305. Patient had a blow in his right side of face and he had
ecchymosis of buccal sulcus and can't open his mouth:
a. Lefort I
b. Lefort II
c. lefort III
d. Zygomatic fracture****
308.306. Most used scalpel in oral surgery:
A. Number 2 blade.
B. Number 6 blade.
C. Number 10 blade.
D. Number 12 blade.
E. Number 15 blade.****
309.307 -Intercoronal bleaching by :
a-Bromide
b-H2o2
c- sodium perborate***
310.308. -Patient who has un-modified class II kennedy classification,
with good periodontal condition and no carious lesion the best clasp
to use on the other side ( teeth side ):
a) reciprocal clasp.
b) ring clasp.
c) embrasure clasp.****
d) gingivally approaching clasp.
311.309. -in maxillary inlay dentist do reverse bevel why:
1-retention****
38

2-resistance
3-pervent lateral displacement
312.310. Shape of rest seat in R.P.D :
1. Spoonshape and square***
2.Concave
3.Convex
313.311. - Lateral ptrygoid muscle arises from :
1)medial surface of lateral ptrygoid plate
2)infratemporal surface of sphenoid and lateral surface of lateral
ptrygoid plate***
3)TMJ capsule and TMJ disc
314.312. Bone between 2 roots is parallel to :
A gingival margin****
b pdl
c tooth apex
: All of these are ways to give L.A with less pain .313

.315

.Give it slowly -1
.Stretch the muscle -2
.Topical anesthesia -3
.The needle size over than 25 gauge -4
****A 1 ,2,3
B 1,2,3,4
C-1,3
D- 2,3,4
316.314. Shoulder finishing line is the choice for porcelain because :
a- It provides enough thickness .****
b- Make the margin of restoration obvious in impression.
c- Assist the technician to locate the finish line on the die.
d- its advantages during setting and casting .
? Ca(oh)2 between appointments . why - .315

.317

Presence of parents in dental visit with child 3 or 4 yrs. - .316


:encourage
**** A. positive behavior
B. negative behave
C. no difference in both
39

.318

:Clicking at posterior sides in denture .317


A. inappropriate occlusal plan
****B. high vertical dimension

.319

: Directly Observed Therapy (DOT) for the Treatment of .318

.320

. HIV ****Tuberculosis Cardiac diseases Diabetes :Type of brushing bristles works as active part .319

.321

a. Stillman's technique
**** B. bass technique
c. Roll technique
Using a larger file while reducing the length in endodontics is - .320
:called
. A.Crown down
****.B. Step back
: internal resorption apear in X-ray as - .321

.323

. a- R.L around apical area


**** b- enlarged R.L in the root
. c- localized radiopaque in the root
d- radiopaque around apical area
: Radicular cyst is related to - .322

.324

A vital teeth ****. B non vital teeth : Patient came to clinic nd you found during examination .323 .325
multiple sebaceous glands at the back of the neck , osteomas in
.mandible , supernumerary teeth and impacted teeth
: Your diagnosis is
A - Cledocranial dysplasia
40

.322

****B Gardner syndrome


. C-paget's disease
D - Ectodermal dysplasia
Best diagnosis of root fracture involved in place of fracture during .324

.326

:healing
A. interproximal connective tissue
****B. interproximal bone
C. interproximal connective tissue and bone
D. inflammation tissue
327.325. -Since composite tooth preparation should be conservative so
the design:
a-Amalgam in moderate and large cavities.
b- Beveled amalgam margines.
c-Conservative restorations.****
328.326. - diabetic patient and allergic to penicillin with swelling dueto
caries :
1-give clindamycine then RCT
2-RCT and erythromycin ****
3- RCT and antibiotic
329.327. Maryland bridge :
A. Used for 2 missing teeth
B. For young patient with good oral hygiene ****
330.328. Laser core can used in curing of composite :
A- Aragon .****
B - Hene .
C - Np(yag) .
331.329. Mesiodistal width of gingival seat of classII amalgam
restoration is :
A. 1mm
B. 0.5 mm
C. 1.5mm **
D. 2 mm
What is the name of laboratory test of HIV positive .330

.332

****patients:Elissa
333.331. - Polyether impression materials:
a. Are less stable dimensionally than polysulfide rubber.
41

b. Are less stiff than polysulfide rubber.


c. Can absorb water and swell if stored in water.****
334.332. horner's syndrom affect:
a-sympathetic nerve supply of pupil****
b-asympathetic nerve supply of iris
c-asympathetic nerve supply of .
335.333. Amount of 2% xylocain in carpool
A..001 mg
B.002 mg
C 36 mg***
336.334.In middle face fracture all except
1. Bleeding from nose
2. Double vision
4. Hematoma on floor of the mouth****
337.335.All true except about Thyroglosal tract cyst
1. Midline
2. Any way alonge the thryro glosal duct
3. Base of tounge
4.not present in hyoid bone****
338.336.SSC all except
1. Select the smallist size
B. Sharp line angle should rounded
3. Sharp smooth margian with tooth contour
4. 2_3 m subgingival****
The labail mechanical lock disadvantage in class III composite .337
( ) restoration
***Increase retention .a
B. Esthetics problem
C. Used for packing of filling
***D. loss of contact with adjusent teeth
We are not sure from the answer because the question is not
.complete and here they are both right
MTA used with.338 .340
a-vital teeth only
b-non vital apexcification
****c-vital and non vital pulp therapy
d-with anterior teeth
42

.339

The lagerist concentration of fluoride in .339

.341

Tooth paste .1
****Flourid varnish .2
Flouridated tooth paste .3
Flouridated milk.4
after removal of ortho white spots found ttt is .340 .342
a_night guatd with vital bleaching
b_venners
c_micro abrasive****the right answer should be this but with fluride
application
d_walking bleachin
criteria of ANUG.341

.343

herps simplex criteria of.342

.344

Faliure of local anesthesia may be due to all of the following .343


except
****A. Type of anesthic solution
B. Dentist tech
C. Acute infection
( ) D. Collateral inervation
Not used as root canal filing in primary teeth .344
***reinfoced ZnO/E ,1
ZnO/E .2
Vitapex .3
KRI paste .4
Vertical hieght.345 .347
A. Uper from hair line to tip of nose
****B. Middle from tip of nose to base of lower lip
C. Lower from base of lower lio to chin
D. Three parts not equal

43

.346

.345

dis lnfection of gp.346 .348


a_autoclave
b_dry oven
****c.sodium hypo chlorite
in class 2 amalgam width of gingival floor mesio distal .347
a_.5
b_1
****c_1.5
d_2 mm
y pregnant woman has red shiny solitary nodule 348.27
tend to bleede by probing
a_hemangioma
****b_pyogenic granuloma
c_giant cell granuloma
d_lymfoma

.349

.350

Regarding Asthmatic pt and NO2 .349 .351


a. In mild and moderat asthma NO2 is contraindications
****B. in sever asthma NO2 IS contraindicated
C. Bring inhalator after sedation
352.350.critical ph of saliva at which
demineralization of enamel begins
a_6.5
b_5.5****
c_7.5
353.351.Consciouse sedation in oral cavity
a_LA
b_general a
c_intra venous....
d_nitrogen oxide****
354.352. Not with topical anestheti
1. 10_15 second not effective
2. Decrease pain of needle
3. Spread as wide area as possible
355.353. To prevent diffusion hypoxia after NO2 we give oxygen for
A. 2_3 mint
B. 4_6 mint****
44

C, 30 mint
D. 60 mint
356.354 Concentration of NO2 to O2
A. 50_70 NO2 AND 30_50 O2
B .30_50 NO2 70_ 90 O2****
357.355.the main cause of analgam faileure is
a_improper cavity prepration****
b_saliva contamination

358.356.CaOh is
a-highly acidic
b-highly alkaline****
c-neutral.
359.357. Radiolucent lesion on hard palat may be one of following
except
A. Nasopalatian cyst
B. Incisive canal cyst
C. Truma
D. Ranula**
360.358. epilepsy parapalsy mean?
361.359. uses of cephalometeric?
362.360. the most favorable condition from class 1 occlusion
relationship
1-mesial step***
2- distal step
3- flush terminal plan with developmental spaces
363.361. function of dentin prime
a_smear layer
b_increase surface energy ***
c_dentin etch
364.362.factors precipitate sickl cell anemia crisis, The answer is Cold
infection dehydration and ischemia
365.363. Sequence of primary teeth eruption
1. A b d c e*****
2. A B C D E
3. C A B D E
4. C B A D E
366.364.In countries with higher annual population growth rates, the
need for community based preventive programs would be greater
for:
OR
367.Communities with high annual population growth need education
about:
45

i. Dental caries. ***


ii. Periodontal disease.
iii. Dentofacial anomalies.
iv. Dental fluorosis
368.365.maxilarry growth
a. Endochondral
b. Downwad and forward
c. Apposition on nasal floor
d. Apposition on max tuberosity
e. A.1 an 2
f. B,2and 3***
g. C,2,3,and 4
369.366.Cbc test
a. Wbc,rbc,platelet,hb,hct***
b. Wbc,rbc,platelet,hb,hct,ca
c. Wbc,rbc,platelet,hb,hct,ca, cl
370.367.One of these is less exposed to extensive dental caries:
a.Obes, malnourished
b.Pt. has xerostomia
c. Less plaque score. ***
371.368.Gutta percha contains mainly:
a. gutta percha 20%.
b. zinc oxide 66%. ***
372.369.Step deformity of the mandibular body fracture may due to:
a. Forward pull of lateral pterygoid muscle.
b. Upward pull of masseter and temporalis. ***
c. Toward pull of medial pterygoid muscle.
d. Downward pull of geniohyoid and myalohyoid.
373.370. Class II composite resin is lined by:
a. G.I. ***
b. Reinforced ZOE.
c. ZOE with epoxy cement.
d. Cavity varnish.
374.371.Procedure done before applying pit & fissure sealant:
aAcid etch by phosphoric acid. ***
bfloride
375.372. In radiographs, which disease causes multiple radiolucencies:
with multipl bone fructure
a. Hypothyroidism.
b. Hyperparathyroidism. ***
c. Ricket disease
376.373.Traumatically fractured crown of central incisor in an 8-yearsold child with pulp exposure ( more than 1 mm. ) half hour ago,
46

medical history is non- contributory and the tooth is not displaced.


What is your management:
a. Endodontics-pulpectomy and obturation.
b. Direct pulp cap with caoh and composite.
c. Caoh pulpotomy.
d.Total extirpation of pulp and caoh. ***
( apexification ).
377.374.After u inject L.A for 2nd. max. molar pt. becomes colorless
with external swelling its due to :
a. 1/ facial artery.
b. 2/ plexus vein. ***
c. 3/ Posterior alveolar nerve.
378.375.Ugly duckling stage:
a. 9-11 years old.***
b. 13-15 years old.
c. 7-9 years old.
379.376.Avulsed teeth stored in:
a. Milk. ***
b. Water.
c. c.saliva
380.377.A child patient undergone pulpotomy in your clinic in 1st
primary molar. Next day the patient returned with ulcer on the right
side of the lip, your diagnosis is:
a. Apthosis.
b. Zonal herpes.
c. traumatic ulcer.***
381.378.pt. came to dental clinic having a haemological problem after
lab test they found that factor VIII ( 8 ) is less 10 % whats the
diagnosis:
a.
Hemophilia A. ***
b.
Hemophilia B.
(Defect in factor IX (9) : hemophilia B = Chrismas Disease).
382.379.A 55 years old patient with multi-extraction teeth, after
extraction what will you do first:
a. Suturing.
b. Primary closure should be obtained if there is no sufficient tissue.
c. Alveoplasty should be done in all cases. ***

383.380.To hasten Zinc oxide cement, you add:
a. Zinc sulfide.
b. Barium sulfide.
c. Zinc acetate. ***
d. Barium chloride
47

384.381.8 years old child came to your clinic with trauma to upper
central incisor with pulp exposure and extencive pulp bleeding your
treatment will be:
a. direct pulp capping.
b. pulpectomy with gutta percha filling.
c. apexification.
d. pulpotomy with calcium hydroxide. ***
385.382.8 years old child suffered from a trauma at the TMJ region as
infant complaining now from limitation in movement of the mandible.
Diagnosis is:
a. Subluxation.
b. Ankylosis. ***
386.383. Safe months to treat pregnant ladies:
a. 1 - 3
b. 4 - 6. *** (2nd trimester)
c. 7 - 9.
387.384.Mandibular 1st permanent molar looks in morphology as:
a- primary 1st mand. molar.
b- primary 2nd mand. molar. ***
c- primary 1st max molar.
d- primary 2nd max molar
388.385.White lesion bilaterally on cheek & other member in the family
has it:
a. leukoplakia.
b. white sponge nevus. ***(Cannon's disease) Hereditary disease
c. others.
389.386.What type pontic design would you do in a patient with a high
esthetic demand when preparing teeth number 9 11 for a F P D:
aRidge lap or saddle pontic.
bAn ovate pontic.
cModified ridge lab pontic. ***
Dental Decks - page 483
390.387.Child has blue swelling on gingiva with no sysmptoms just
heavy saliva, what is ur diagnosis:Eruption hematoma.***
391.388.Hemisection means:
a. Surgical removal of the apical portion of the root.
( Apicectomy ).
b. Removal of one or more roots. ( Amputation ).
c. The root and the crown are cut lengthwise. ***
d. None.
392.389.What is the name of the instrument used to diagnose halitosis:
Halimeter. ***
48

393.390.Hyperparathyrodism : Multiple fractures , multiple


radiolucencies at the angle of mandible , usually associated with
Renal Failuer .
394.391.Congenital epulis of newborn : mostly at maxillary anterior area
, it interfere with feeding or respiration and requires excision and
rarely recurrent 8:1 in females.
395.392.Treacher Collins : deformity in zygoma , mandibular
Retrognathia , Clift palate and eye drop down , ear deformity with
hearing loss & open bite ( but he is mentally normal )
396.393.Complete epthlilization occuers after 7-14 days , but complete
maturation needs 6 months after periosurgery
397.394.Upper E DL cusp is the sharpest cusp in both dentitions ,and is
the largest in primary
398.395.Upper 6 ML cusp is the sharpest cusp in permenant dentition
and is the largest DL cusp.
399.396.1st sign of calcification occuers 14 weeks Intra uterine while
first formation of tooth bud is 8 weeks intra uterine.
400.397.sodium hypoclorite house hold percentage
a. a.5.25***
b. b.0.5
c. c.1.5
401.398..The periodontium comprise which of the following tissues:
Gingiva and the PDL.
Gingival, PDL, and alveolar bone.
Gingival, PDL, alveolar bone, and cementum. ***
Gingival, PDL, alveolar bone, cementum, and enamel.
402.399. Communities with high annual population growth need
education about:
Dental caries. ***
Periodontal disease.
Dentofacial anomalies.
Dental fluorosis.
403.400..An examination of the edentulous mouth of an aged Pt. who
has wore maxillary complete dentures for many years against six
mandibular teeth would probably show:
A.Cystic degeneration of the foramina of the anterior palatine nerve.
B.Loss of osseous structure in the anterior maxillary arch. ***
C.Flabby ridge tissue in the posterior maxillary arch.
D.Insufficient interocclusal distance.
404.401..water fluoridation?
a. 0.5 0.8 mg\liter ***
b. 0.2-0.5 mg\liter
c. 2-3 mg\liter
49

d. 1-5 mg\liter
405.402..Hand instrument which we used to make internal angles
retentive grooves and preparation of cavity walls in the cavity is:
a.Angle former. ***
(To shape line and point angels inter a cavity giving retention)
406.403.patient came to dentist after previous stressful procedure
complaining of burning & discomfort of his lip on examination u found
lesions on the palate, diagnosis is?
a. A.contact dermatitis allergy
b. B.aphthous ulcer
c. C.herpes simplex ( herpetic gingivostomatitis ) ***
407.404..has been proven that amalgam restoration has the following
characteristics:
a. Micro leakage decrease with aging of the amalgam restoration.
b. It is the least techniques sensitive of all current direct
restorations.
c. High dimensional changes.
a, b and c.
a and c.
a and b. ***
b only.
408.405..Plaque consists of?
a. Bacteria ****
b. Inorganic material
c. Food
409.406..Eruption cyst "Eruption hematoma" can be treated by?
a. A.No treatment. ***
b. B.Immediate incision.
c. C.Complete uncoverage
d. D.Observe for one week then incise
410.407..gamma phase 2?
a. A.2% copper
b. B.4% copper
c. C.10 % copper
d. D.13 % copper***
411.408..making impression of an oral cavity is: The best material for
taking impression for full veneer crowns?
a-Irreversible hydrocolloid (Alginate) (the least accurate imp. mat.).
b-condensation type silicon.
c- polyvinyl siloxanes. (Addtional silicon type)***
d-poly sulfide.
412.409..The rationale for pit-and-fissure sealants in caries prevention is
that they:?
50

a. A.Increase the tooth resistance to dental caries.


b. B.Act as a barrier between the sealed sites and the oral
environment.****
c. C.Have anti-microbial effect on the bacteria.
d. D.None of the above answers is correct
Maybe B ill make sure
413.410..Teeth that have lost pits and fissure sealant show?
a. A.The same susceptibility to caries as teeth that have not been
sealed.
b. B.Higher susceptibility than non sealed teeth.
c. C Lower susceptibility than non sealed teeth****
d. D.The same susceptibility as teeth with full retained sealant.
414.411..action of autoclave?
1.steam and then overheat overheat of steam***
2.steralization by disifection at high temp.
415.412.best describe reparative dentine
1. Occur in pulpal wall due to local irritation or inflammation***
2.same like secondary dentine
416.413..Organism that initiates caries:
a. a.Streptococcus mutant. ***
b. b Streptococcus salivarius.
c. c Spirochaeta.
d. d.e-coil. Skin bacteria.
N.B or bacteria didnt present in infants mouth also same answer
417.414..The most common benign tumor in oral cavity is:
a. a.Fibroma. ***
b. b.Papilloma.
c. c. Lipoma.
:Difference between Gracey and universal curette ..415

.418

.A.Section of gracey is hemicircular and in universal triangular


.B.Gracey has one cutting edge while universal has two
C.Gracey used for cutting in specific area while universal is in any
.area
.D. Universal 85 bevel, gracey 70 offset
a and d
.a, b and c
.b, c and d
***b and c
51

?:Cause of fracture of occlusal rest ..416

.419

.a.Shallow preparation in marginal ridge

.a

.b Extension of rest to central fossa


c Improper centric relation
***d.all

.c

.d

minimum thickness of veneer..417


a 0.3

.420

.a

*** b.0.5
c 0.7

.b

.b

.c

421.418.. Geographic tongue is always accompanied in patient with:?


a. a.Diabetes.****
b. b Erythema multiform.
c. c Iron deficiency.
N.b. It is maily asscosiated with psoriasis . Diabetetes and
psychological stresses are risk factors
422.419..Solitary bone cyst management:?
a. Anti-inflammatory and follow up.
b. Curettage and close. ***
c. Marsupialization and antibiotic
d. No active management
423.420..Endomethasone is a root canal sealer that: ?
a. a.Dissolves in fluid so it weakens the root canal filling.
b. b.Very toxic containing formaldehyde.
c. c Contains corticosteroids.
d. d All***
424.421..What kinds of radiographs which we do not use for TMJ
movements?
aTranscranial. ***
bComputerized tomography.
cConventional tomography.
dArthrography.
425.422..Squamous cell carcinoma is found in :?
52

a. Epithelial oral tissue or mucous membrane. ***


b. Connective tissue..
c. oral mucus membrane
426.423.. Osteogensis during endodontic surgery aimed to prevent:
a. fibrous in growth. ***
b. growth factor.
c. formation of blood.
427.424..Cracked tooth syndrome is best diagnosed by?
a. Radiograph.
b. b. Subjective symptoms and horizontal percussion.
***
c. c. Palpation and vertical percussion.
d. d. Pulp testing.
428.425..Drug used to increase or stimulatesaliva is: ?
a. anticholinergic.
b. cholinergic. ***
c. antidiabetic.
d. anticorticosteroid.
429.426.. Sharp pain is due to which type of fibers?
a. A fibers. ***
b. B fibers.
c. C fibers.
430.427..Secondary dentin occurs due to:?
aocclusal trauma.
brecurrent caries.
cattrition dentin.
dall of the above. ***
431.428..root perforation: mta
432.429..How do you know if there are 2 canals in the same root: OR
Best way to detect presence of 2 canals?:
a. Radiographically with 2 files inside the root. *** (putting 2 files &
take X-ray)
b. The orifices are close to each
433.430..The rows show truth, the column shows test result:
a. Cell A has true positive sample.***
b. Cell A has true negative sample.
c. Cell A has false positive sample.
d. Cell A has false negative sample.
434.431..Streptococcus mutans cause caries & this disease is?
a. epidemic. 2 /
b. B.endemic. *** 3 /
c. C.isolated.
53

435.432.. intruded tooth ideal time of splint: ?


a. 1/ splint (7-14) days.***
b. 2/ or (3- 14) weeks.
436.433.. Safe months to treat pregnant ladies:?
a. 1 - 3
b. 4 - 6. *** (2nd trimester)
c. 7 - 9.
437.434..What the influence of xylitol: ?
a. A.It causes caries.
b. B.Safe to the teeth.((reduce caries)) ***
c. C Increase saliva.
d. D.Decrease saliva.
438.435.. The infection will spread cervically in infection from:?
a- lower incisors.
b- lower premolars.
c- lower 2nd and 3rd molars. ***
d- upper incsisors.
439.436..forceps to remove eppolus fessuratum allis
440.437..Pt. with renal dialysis the best time of dental ttt. is:?
a1 day before dialysis.
b1 day after dialysis. ***
c1week after dialysis.
441.438..Epileptic patient you will not give him :
1. Aspirin.
2. Azoles.
3. Metronidazole. *** ( Flagyl ).
4. All of the above
442.439..Patient presented to you after fitting the immediate denture 5
10 months, complaining pain and ulcer inflamationin the
mandibular, what is the diagnosis: ?
1. Epulis fissuratum. ***
2. Hypertrophic frenum.
3. Pappilary hyperplasia
443.440.. tissue response to oral hygiene instructions is detected by:?
aProbe pocket depth.
bLess bleeding on probing ***
c..less inflamation of gingiva
444.441..The best definition to odontoblast: ?
aIts subadjacent to predentine & odontoblastic process. ***
bOdontoblast cell is more in the cellular pulp than radicular.
445.442..Wax inlay which type contain in much gradient: OR The kind of
onlay wax used in cast:
a. Paraffin wax. ***
54

b. Bee wax.
446.443..Two weeks baby born with 2 anterior teeth which is highly
mobile, and his mother have no problem or discomfort during nursing
him what is ur management? :
a. do not do anything as the baby have no problem during feeding.
b. do not do anything as the mother don`t feel discomfort.
c. U must extract as soon as possible to avoid accident inhalation of
them. *** (because those are natal teeth)
d. Do nothing, it will shell by it self
447.444..Edentulous pt. class II kenndy classification 2nd premolar used
as abutment when we serving we found mesial undercut what is the
proper clasp used:?
a. 1/ wrought wire with round cross section. ***
b. 2/ wrought wire with half round cross section.
c. 3/ cast clasp with round cross section.
d. 4/ cast clasp with half cross section.
448.445..Medicine ethics aim to:?
a. A The dentist should study to know patient psychology.
b. B Not to compromise or undermine ability to treat patient in
community as professional.
c. C Ability to make decision.
d. D All of above. ***
449.446.. You should treat ANUG until the disease completely removed.
Otherwise, it will change to necrotic ulcerative perio.:
a. Both sentences are true. ***
b. Both sentences are false.
c. 1st true, 2nd false.
d. 1st false, 2nd true.
450.447..Premature contact between upper and lower ant. teeth in
eccentric occlusion while there is absolutely no contact on the centric
occlusion. So the management is by grinding of?:
a. incisal edge of ant. max. teeth.
b. Incisal edge of ant. man. teeth.
c. Inclination of Ant. max. teeth lingually. ***
d. Inclination of ant man teeth.
451.448..which part of root canal diameter is the smallest: OR What is
the smallest area in root canal:?
a. A Radigraphical apex.
b. B Apical foramin.
c. C Apical constriction. *** ( Cementodentinal junction )
d. D Cementoenamel junction.
452.449.. Salivary gland disease (tumor) with perineural invasion
1 :. ?
55

a. 1.pleomorphic adenoma.
b. 2. Adenocyctic carcinoma. ( Adenoid cystic carcinoma ). ***
453.450..child with trauma and swollen lip for the last few days, no pain
and teeth are good, what will u do next?:
atake an x-ray of the lip to exclude any foreign object.****
bcold packs to manage the swelling.
canti inflammatory drugs.
454.451..which of the following events occurs during dentinogenesis:?
a. Odontoblasts become long cuboidal. ***
b. The matrix and proteoglycans maturate with collagen fibers.
455.452.During 3/4 crown preparation on premolar, bur used to add
retentive grooves?
a. a.Radial fissure.
b. b. Tapered fissure***
456.453..intruded tooth minimum discolration x.ray root resorption
A.inflamatory resorption
457.454.Pt came to clinic for construction of max complete denture
during intraoral exam u found severe anterior undercut and
posteriorly lateral to max tuberosity what should dentist do?:
a. reduction of max anterior undercut and relief of post one
b. reduction of ant and post undercut ****
c. reduction of post and relief of ant
d. relief of both
458.455..Epistien pearls ...>>>mid palatine raphe.
459.456..gold onlay .pain on chewing only ...>>>>.occlusal
interferance affect periodontal tissue
dentist decide to make indo but patient fainancial status not ..457

.460

.....good..persue him if refuse wright that in chart and extract


..
Child have dental caries in 3 or 4 surfaces of his 1st primary ..458
:molar we will replace them with
*** preformed metal crown
porcelain crown

.b

amalgam crown

.c

composite crown

.d

.a

56

.461

Q about tissue retraction cord...in files ..459


amalgam character ....in files ..460

.462

.463

year old child received a trauma to central incisor few hours 10 ..461
ago. He has pain and numbness in gingiva around tooth. there's no
?:mobility or displacement of tooth
***give oral analgesics and follow up
RCT

.464

.a

.b

extract tooth

.c

splinting followed by pulpectomy

.d

condylar fructure in left ..deviation to left ..462


best inmp material .poly venyle ..463

.465

.466

bilateral infection in submental , submandibular and sublingual .464


? triangles with difficulty in breathing and swallowing is
Chronic periapical abscess
*****Ludwig angina

.1

.2

468.465 Whats the access shape of lower 7 ?


a. Rombiod ***
b. Converge
c. Traiangle base distally
d. Traiangle base buccally
469.466 which muscle inserted in tmj capsule ?
a. Temporalis
b. Masseter
c. Medial pterygoid
d. Lateral ***
470.467 which artery from external carotid artery supply the face ?
a. Occlomotor
b. Occipital
57

.467

c. Facial***
471.468 main action of aspirin is
a. Platelet aggregation***
b. Prothrombin
c. Fibrenogin
d. Thromboplastin
472.469 whts the age of ortho treatment ?
a. A 7 ***
b. B 6
c. C 8
d. D 9
473.470 most tooth congenital missing ?
a. A upper 2
b. B upper 4
c. C lower 3
d. D lower 8 ****
474.471 most common cell in granuloma
a. A lymphocyte***
b. B plasma cell
c. C neutrophils
d. D macrophage
475.472 treatment of odonogenic keratocyst ?Surgical excision
476.473 bonding of enamel ?
a. A chemical
b. B mechanical
c. C Micromechanical****
477.474 during ext of upper 8 it intruded posterior whats the
manegment ?
a. A leave it
b. B leave it until 3 weeks then extract ****
c. C extract immediate by g . a
478.475 Patient come to clinic wants to construct lower denture during
the examination you find wide and shallow frenum what is the
suitable ttt before denture:
a. Z-plasty
b. Sub epithelial incision
c. Mucous incision
d. supraperiosteal incision of vestibuloplasty ***
479.476 partial denture. Pt came to you need upper and lower complete
denture after examination you noticed that he has an anterior
undercut + undercut in post. area near to tuberosity, what you will
do:
a. remove undercut from anterior area
58

b. remove undercut from post. area


c. remove under from post. Area & make reline in denture for
anterior area
d. remove under from ant. Area & make reline in denture for post
area
480.477 . gravity force Female first time need denture you do
examination to soft and fault, you said to pt. the denture will resist
lateral movement but vertical movement cause loading (removing) of
denture what is type of vault:
OR
481.Old patient come need complete denture, after examination the
dentist told him, lateral forces will be good but vertical forces will
break the seal:
a. U shaped palate
b. V shaped palate***
c. flat
d. paralyzed
482.478 An examination of the edentulous mouth of an aged Pt. who
has wore maxillary complete dentures for many years against six
mandibular teeth would probably show:
a. Cystic degeneration of the foramina of the anterior palatine
nerve.
b. Loss of osseous structure in the anterior maxillary arch. *
c. Flabby ridge tissue in the posterior maxillary arch.
d. Insufficient interocclusal distance
483.480 Patient who has un-modified class II kennedy classification, with
good periodontal condition and no carious lesion the best clasp to use
on the other side ( teeth side ):
a) reciprocal clasp. ( aker's clasp ).
b) ring clasp.***
c) embrasure clasp.
d) gingivally approaching clasp.
484.481 Pt. construct for him a complete denture after few days he
came to u complaining from pain & white spots on the residual ridge
do relief in that area & give him ointment & after few days he came
again complaining the same but in another area the main cause is :
a. Uneven pressure on the crest of alveolar ridge. ***
b. Increase vertical dimension
485.482 parotid duct is opposite to:
a. maxilary premolar
b. maxilary 1st molar
c. maxilary 2nd molar ***
d. mandibular 1st molar
59

486.483 Pt. 8 yrs. old fracture upper central 9 months ago with-ve
vitality test best treatment:
a. endo
b. b. Pulpotomy with formocresol
c. c. Pulpotomy with caoh
d. d. Total debridement + apexification ***
487.484.In primary tooth for restoration before putting the filling u put:
a. base.
b. calcium hydroxide.
c. varnish.
d. you put the filling after proper cleaning and drying. **.
488.485.What is the name of the instrument used to diagnose halitosis:
Halimeter.
489.486. The narrowest canal found in a three root maxillary first molar
is the:
a. Mesio-buccal canal. ***
b. Disto-buccal canal.
c. Palatal canal.
d. Disto-palatal canal.
e. Mesio-palatal canal.
490.487 Fractured tooth to alveolar crest, what is the best way to
produce ferrule effect:
A. restore with amalgam core sub-gingival.
B. crown lengthening
C. extrusion with orthodontics ***
491.487 Bonding agent EBA consist of:
OR
What does Enamel bonding agent (EBA) consist of:
a. Unfilled resin ***
b. primer and bonding agent
c. A mixture of resins in an acetone or ethanol solvent
d. A wetting agent and resins
e. A mixture of priming and bonding agents
492.488. The most ductile and malleable metal is:
a) Sliver.
b) Gold .*** .
c) Copper.
d) Platinum.
493.489 Wax patterns should be invested soon because:
a. The wax exhibits shrinkage. ****
60

b. Becomes brittle.
c. Subjected to fracture.
The wax has memory and begins to distort
494.490.What type of pontic you will use in premolar area that will give
illusion and clearance:
i. Modified ridge lap****
ii. Ovate
495.492 Most retentive abutment:
1. molar with diverge root ***
2. Molar with conical root
3. Molar with little interseptal bone
496.493 3 years old pt., water fluoridation 0.2 ppm what is the
preventive treatment:
a. 0.25 mg. fluoride tablet. ***
b. 1 mg. fluoride tablet.
c. Fluoridated mouth wash.
d. Sealant.
497.494 To hasten Zinc oxide cement, you add:
a) Zinc sulfide.
B) Barium sulfide.
C) Zinc acetate. ***
D) Barium chloride
498.495 Sterilization in autoclave:
a. 20-30 min at 121c
b. 2-10 min at 134 c
c. A+B***
499.496.Pt. child we put restoration after:
i. Base
b. caoh
c. varnish
d. just clean with water and dry****
500.496 A 7 years child has a habit of finger or thumb sucking what is
the management or best way to start a therapy with:
a. Rewarding therapy.
b. Counseling therapy. ***
501.Punishment.
d. Remaindering therapy.
e. Nothing
502.497 What is the color complex of calculus present in 18 y pt.:
a. A black
b. B. brown
c. C red
61

d. D purpul***
503.498 Sharpening the curette and sickle, the cutting edge should be
at angle:
a- 50-60
b- 70-80. ***
c- 80-90
d- 60-70
504.499 Contraindication of gingivectomy:
a- periodontal abscess
505.500 Best Implant Success in bone:
a. type1
b. type2 ***
c. type3
d. type 4
506.501 Long question For patient has fracture of condyle what is the
most suitable graft taken from:
a. anterior iliac graft
b. posterior iliac graft
c. post chondra graft
507.502.A minimum of two points of fixation should be used to provide
stable internal fixation of mandibular symphysis fractures:
a. Apply the first plate to the inferior border of the mandible.***
b. Use a 1.5 mm drill bit with 6 mm stop to drill monocortically
through the plate hole next to the fracture
c. B. Apply the first plate to the inferior border of the mandible.
d. Use a 1.5 mm drill bit with 6 mm stop to drill biocortically
through the plate hole next to the fracture.
e. Insert a 2.0 mm screw, 6 mm in length a 2.0 mm screw, 6 mm in
length
508.503 A child has an anterior cleft, this happened when:
a. 4-6 weeks intrauterine
509.504 Electrocyte disturbances during massive and rapid blood
transfusion:
1.
Hypernatremia
2.
Hyperkalemia***
3.
Hypercalcemia
4.
Hyponatremia
510.505 pt with complete PD complaint of upper lip too long
a. Deficient vit b
b. Defect in vertical dimension***
c. Ant upper teeth short
511.506. pt hv facial divergence
a. Class 1
62

b. Class 2
c. Class3 ******
512.507.The 4th canal in maxillary first molar found in :
i. In palatal root
ii. In MB root ***
iii. In DB root
iv. Between palatal and distao-buccal c
513.508 Pt. has upper complete denture and lower anterior, what
happen upper :
i. Upper loss bone anterior maxilla
ii. Flapy ridge***
514.509 Caries in endemic it's means :
a. Affect small population
b. Has affect on countries with numerates***
c. Present in area or region naturally
515.510 How to ttt fracture of symphysis by :
a. Put plate infra coronal bone mono cortical screw ***
b. Put plate infra coronal bone double cortical screw
c. Put plate supra coronal bone mono cortical screw
d. Put plate supra coronal bone double cortical screw
516.511.2y pt. with spot facial surface what to do ttt plane :
a. Parent counseling
b. Anticipatory guidance
c. Nutritive analysis***
d. Caries examination
517.512 Extra oral INF alveolar injection pt. truismus inject lateral :
Upward + post***
518.513- 5 y old did extraction , after few days came with fever , sore
throat ,cervical lymphoadenopathy , pain , bleeding ,investigation
show anemia ,thrombocytopenia WBC low :
a. Acute lymphoblastic leukemia ***
b. Acute myeloanemia
c. Chronic myeloanemia
519.514 school going pt with severe overjet dentist did lateral
cephalometric and found SNA 80 SNB 82 ANB -2
a. A cl 1 malocclusion with cl 2 skeletal
b. B cl 3 mal with cl 2 sk
c. C cl 3 mal with cl 1 sk
d. The correct answer is Class 1 dental or (mal), 3 skeletal. Maybe
the one who wrote the question forget about the choices.
520.515. Teeth that have lost pits and fissure sealant show:
a. The same susceptibility to caries as teeth that have not been
sealed.
63

b. Higher susceptibility than non sealed teeth.


c. Lower susceptibility than non sealed teeth. ***
d. The same susceptibility as teeth with full retained sealant.
521.516.Success of pit & fissure sealants is affected mainly by:
1) increased time of etching
2) contamination of oral saliva. ***
3) salivary flow rate.
4) proper fissure sealant.
522..517 Scallopped border above inferior alveolar canal between roots
of mandibular molars, this lesion is:
a) Solitary cyst.
b) Aneurysmal bone cyst.
c) Traumatic bone cyst. ***
523.518..use of caoh interappoinmet
Ans. Antibacterial ***
524.519. Dentinogenesis imperfecta due to disturbance in which stage
A.initiation
B.histodifrrentiation***
C.morphodifferentiation
D.proliferation
525.520. During modified widman flap third incision by
A.orbans knife***
B.bp 12
C. Bp 15
526.521.previous Rcf exposed to oral environment within how many
time must retreated
A.2 months
B.6 week
C.2 weeks
D.six days
527.522.for IMF what u use
A.4 inch and 22 guage
B.4 inch n 28 guage
C.6 inch 20 guage
D.6 inch 26 guage****
528.523.unilateral crossbite due to
A.unilateral restricted growth of maxilla
B.bilateral restricted growth of maxilla
C.atypical growth of mandible ***
D.Unilateral restricted growth of mandible
529.524. To measure vertical dimensions
Ans is Willis guage***
64

530.525. To retract flap with cheek


Ans is Minnesota retractor****
531.526. Disadvantage of full thickness mucoperiosteal flap
Ans may be Integrity of papilla lost
532.527. Pt want to increased keratinised gingiva which surgery
.apical displaced *****
Coronal displaced flap
533.528. Fixation of 2mm plate which drill u use
A.2.5 mm
B.2 mm
C.1.1mm
D.1.5 mm****
534.529. Temperature during bone cutting which is dangerous to bone
A.42 degree for 30 sec
B. 42 For 1 min
C.46 degree for 39 sec
D.46 for 1 min****
535.530. More flexible
A.k file
B.h file
C.broach
D. Reamer****
536.531. Mechanical exposure in child .pt primary tooth during class 1
A.pulp capping
B.pulpotomy***
C.pulpectimy
D.extraction
537.532. Eruption of permanent 2nd molar
A.8 yr
B.10 yr
C.12 yr***
D.14 yr
538.533. First sign of calcification of any tooth I U
A.14 week***
B.16 weeks
C.6 weeks
D.8 weeks
539.534. Permanent first molar resembles
Ans is primary lower second molar
540.535. Edta used for
A.calcified tissue***
B.pulp tissue
C.debris
65

541.536. EBA contains


Ans is Unfilled resin***
542.537. Oral lesions of lichen planus usually appear as:
a. White striae. *** " Wickham striae"
b. Red plaque.
c. Shallow ulcers.
d. Papillary projections.
e. Builae.
543.538. What is the number of pharyngeal "branchial" arches:
a. 4.
b. 5.***
c. 6.
d. 7.
544.539. Hand over mouth technique is used in management of which
child:
a. Mentally retarded.
b. Positive resistance.
c. Uncooperative.
d. Hysterical. ****
545.540. What are the dominant type fibers found in cementum?
a. a.longitudinal
b. b.circular
c. c.Sharpeys fibers****
546.541.3yrs old child came to the clinic with his parents has a fluctuant
in the mid line of the palate?
a. a.Bohns nodule
b. B.Epistien Pearls****
547.542.apatient had a classII amalgam restoration the next day he
come to clinic with discomfort at the site of the restoration
radiographicaly an overhanging occurs this is due to?
a. a.Lack of matrix usage
b. b.improper wedging***
c. c.no burnishing of amalgam
548.543.patient taking heparin he should do surgery after?
a. a.1 hour
b. b. 2 hours
c. c.4 hours
d. d.6 hours***
549.544.Pacefier habit in a patient of a 7 yrs old age what would you
see in his mouth?
a. a.Anterior open bite, posterior crossbite****
b. b.Anterior open bite with expansion of maxillary bone
c. c.move from incisal to labial
66

d. d.crossbite
550.545. EBA (Enamel Bonding Agent) is :
A) etchant
B) etchant and bonding agent
C)unfilled Resin***
D) priming Agent
551.546 Hybrid layer is :
A) resin dentin interface***
B) bacteria dentin layer
C) composite layer
552.547.Smoothening of facioproximal cavo surface angle in class II
cavity is done using
A) bibeveled hatchet
B) Binangled Chisel****
C)Enamel hatchet
D)chisel
553.548 Patient suffering from severe tenderness at the right auricular
border..pain increases on opening the mouth wide and extreme lateral
excursion leads to deviation of the mandible to the left, however
there is no clicking sounds during the movements.
What should the surgeon expect:
A) anterior Disc displacement with reduction
B)Anterior disc reduction without reduction***
C) Rheumatoid arthritis
554.- Disk derangement with reduction often causes a clicking or
popping sound when the mouth is opened. Pain may be present,
particularly when chewing hard foods. Patients are often embarrassed
because they think others can hear noise when they chew. Indeed,
although the sound seems louder to the patient, others can
sometimes hear it.
-Disk derangement without reduction usually causes no sound, but
maximum opening between the tips of the upper and lower incisors is
reduced from the normal 45 to 50 mm to 30 mm. Pain and a
change in the patient's perception of their bite generally result. It
usually manifests acutely in a patient with a chronically clicking joint;
about 8 to 9% of the time, the patient wakes up unable to open the
jaw fully.) osteoarthritis
555.549.Most common cause of abnormal teeth position:
A)Lack of space or hyperthyroidism
B) oligodontia
C) Hypodontia or lack of space
Oligodontia: also causes abnormal position of teeth but not as
common as hypodontia.
67

556.550.Which one has a positive rake


angle:
A K-file***
B- K reamer
C-protaper
D-profiles
if H file was in the options then it
would be the correct answer. But from
these K-file is the right answer
557.551.Xray showing a mass with R.O and R.L structures at the
deciduous canine lateral area
A- compound odontoma***
B-Complex Odontoma
C-Dentigerous Cyst
558.552-Child 9 years old came with
laceration of the chin and no
symptoms on X-ray we found
bilateral subcondylar fracture. There
was no malocclusion and patient can
perform normal mandibular
movements, ttt of choice is:
A- IMF with rigid wire for 3 weeks
B-IMF using elastics for 6 weeks
C-mandibular stent and restricted movements
D-No treatment and free movements****
559.553-patient who had radio therapy 5 years ago ..what measures
should be taken to prevent osteoradionecrosis
A- extraction with curettage
B)periodontal therapy with smthn??
C)Extraction and hyperbaric oxygen chamber****
560.554-posterior bite plate is used for :
A-posterior movement of anteriors
B- posterior crossbite with anterior deep bite
C-crowding of posterior teeth
D-allows eruption of anterior teeth****
Indications of posterior bite plate :
1.
A posterior biteplate is used to correct an anterior open bite (when
front teeth are not overlapping at all when biting). It is often used with
a functional appliance such as an activator. It does so by allowing the
anterior teeth to erupt (extrude) while preventing the posterior teeth
from erupting (some relative intrusion of the posterior teeth).
2.
It may be used in conjunction with an appliance to expand the palate
(roof of the mouth) to correct a functional crossbite.
68

3.

May also be used to correct a Class I type III (lingually trapped


anterior tooth). It does so by opening the bite hence allowing the
trapped tooth to come into occlusion
561.555-tooth no.11 trauma with crown fracture and RCT, it has a
narrow canal and a post and core ttt is scheduled the choice for post
and core is:
A-carbon fiber post with composite core****
B-cast post and core
C-threaded stainless steel post with amalgam core
562.556-best pulp test using:
A-thermal
B-electric
C-Co2 Endo ice spray****
D-cold
563.557-Mature tooth with extensive
trauma and grey discoloration, no
response to pulp vitality test ttt is:
A- RCT with gutta percha****
B-partial pulpotomy
C-pulpotomy
564.558-Incisor liability is :
A- the 4 primary incisors compared
to 4 permanent incisors****
B-the 4 primary molars compared to
4 permanent molars
C-all the permanent dentition to the
deciduous
565.559-leeway space is:
A-the combined width of C,D,E is less than
the combined width of 3,4,5
B-the combined width of C,D,E is greater than that of 3,4,5***
C-combined width of C.D.E is equal to 3,4,5
566.560-antibiotics prescription is recommended when:
A-diffuse rapid spreading infection***
B-acute localized infection
C-chronic infection
567.561-focal infection theory dictates the spread of infection by :
A-local infection***
B-diffuse infection
sorry cant remember the rest
568.562-most common malignant salivary gland tumor is:
A-mucoepidermoid carcinoma***
69

B-adenoid cystic carcinoma


C-pleomorphic adenoma
569.563-recommended cycle settings for autoclave is:
A- 121 c for 15-20 mins***
B- 134 C for 160-180 mins
C-700 C for 1 hr.
570.564-Ideal impression should have:
A-Long working time short setting time***
B-Long setting time short working time
C-long working time and long setting time
D-short setting time and short working time
571.565- When grooves are placed in an abutment preparation it is for :
A- Anti-rotational purpose***
B-retention
C-stability
572.566-Patient 15 yrs old with history of prolonged tetracycline therapy
and mild yellowish stains treatment includes:
A -Home bleaching***
B-veneers
C-full veneered restorations
D-micro abrasion with pumice
573.567-The most conservative way for teeth whitening is:
A-bleaching****
B-micro abrasion
C- porcelain jacket crowns
D- ceramic veneers
574.568-Grade II furcation involvement treatment of choice:
A- GTR (guided tissue regeneration)****
B- full flap with curettage
C-periodontal therapy
575.569- Healing of periodontal pocket after scaling and root planning
occurs by:
A- long junctional epithelium***
B- fibrous tissue
dont remember sorry
576.570-What is the prosthesis indicated
when loss of teeth is inevitable but
extraction is not recommended at the
present time:
A-interim***
B-transitional
C-conventional
70

577.571-Molar rest seat in RPD design is:


A -concave
B-spoon or saucer shape***
C-concave (yes there were 2 concave)
D-square with sharp angles.
578.572-Diabetic patient who has recently visited the ophthalmologist
and TMJ pain and swelling :
A-Reiters syndrome****
B-hyperparathyroidism
C-rheumatic arthritis
579.573-Nerve supply to the TMJ:
A-facial nerve
B-Auriculotemporal nerve***
C- glossopharyngeal nerve
580.574 The following are true about the maxillary growth pattern :
1-deposition occurs at the nasal floor
2 -deposition occurs at the tuberosity
3-downward forward direction of growth
4- endochondral growth
A-1,2,3***
B-2,3 ,4
C-1,2,4
581.575- Growth of the skeletal system of the fetus arises from :
A-Neural crest cells***
B-mesenchymal cells
C-axio something sorry dont remember
582.576-Best recommended maximum time for re-implantation of
avulsed tooth is:
A-1 hour***
B-20 hrs
C-24 hrs
D-one week
583.577-The PH of ca(oh)2 :
A-10
B-19
C-12.5***
584.578-We give prophylactic antibiotic coverage to which of the
following patients before dental treatment:
A- recent bypass surgery
B-myocardial infarction
C-Patient with Prosthetic valve ***
585.579-Immpression material that has the property of recording fine
details even on wet surface
71

A-Addition silicone
B-condensation silicone
C-alginate
D-polyether***
586.580- Child 4 years old indicated for oral hygiene measures under
general anesthesia , he had a distal caries in lower right D and mesial
caries in lower right E treatment is:
A- Back to back silver amalgam restoration**
B-back to back composite restoration**
C-stainless steel crowns
D-pulpotomy
(I chose amalgam as a filling of choice here as the child is only 4
years old and amalgam will be more durable and better in strength
than composite but ther are small cavitiesso we would also argue
that we can be more conseravative if we use composite and pits and
fissure sealant since these are only proximal cavities I couldnt
find a proper reference to indicate either one unfortunately)
and I guess back to back here refers to the position of the cavities
relative to each other)

587.581-the main component of the floor of the mouth is:


A-tongue
B-digastric muscle
C-Mylohyoid muscle****
D-hyoid muscle
588.582-Child suffered from wheezing , shortness of breath, severe
coughning during dental treatment the cause is:
A-Asthmatic attack***
B- cardiac attack
C-adrenal crisis
D-insulin shock
589.583- Tooth suffering from pain on cold that disappears after
stimulus is removed immediately and there is a crack discovered by
fiberoptics the following represents the case:
A- reversible pulpitis and crown coverage***
B- irreversible pulpitis and MOD amalgam restoration
C pulp necrosis and amalgam restoration
590.584-Maximum strength of amalgam restoration is achieved by:
1- cavo surface angle is 45
2-cavo surface angle is 90
72

3-on sound dentin


4-located in area free of stresses
A-1,2,4
B-2,3,4***
C- all of the above
591.584- The success of intrapulpal anesthesia depends on:
A-back pressure ***
B-L.A agent strength
C-dont remember
592.585- slowest onset of action :
A-lidocaine
B-bupivicaine***
C-mepivicaine
D-prilocaine
593.586-Pulp polyp is:
A-hyperplastic lesion***
B-hypertrophy of the pulp
C- necrotic lesion
594.587- Kidney Function assessment tests include:
A-BUN, Creatinine
B-BUN,Creatinine, K, Ca***
C- K, Ca, CBC
595.588- The signs of gingivitis in heavy smoker Vs. non smoker:
A-same in both
B-less in heavy smoker than Non-smoker***
C-less in Non-Smoker than heavy Smoker
596.589-Communities with high annual population growth need
education about:
A-Periodontal disease.
B-Dentofacial anomalies.
C-Dental fluorosis.
D-Dentla Caries***
597.590-Best instrument to locate vibrating line, it is: A-T-burnisher***
598.591-In normal healthy periodontium the interdental bone is:
A- parallel to CDJ of teeth***
B-perpendicular to the CDJ
599.592-Name of the the periodontal fibers that are immersed in
cementum:
A-sharpeys fibers***
B-horizontal fibers
C-oblique fibers
600.593-Strept. Mutans is:
A-endemic in human beings***
73

B-pandemic
C-epidemic
601.594-Tooth underwent trauma was treated by RCT and was
asymptomatic ,became brownish in color after some time what was
the cause of this discoloration
A-hemorrhage of the pulp in dentinal tubules***
B-debris in the dentinal tubules
C-necrosis
602.595- 48 yrs old woman with Complete denture presented with pain
in different areas and altered taste sensation after performing
examination all was normal with the denture but blood tests show low
levels of ca, hemoglobin, Platelets the most probable diagnosis is :
A-neural problem
B-menopause***
C- hyperparathyroidism

603.596- In Recent years, there has been an evidence that the


prevalence and intensity of the caries attack has been diminished in
the more economically developed countries, mainly because of the
wide spread use of:
A-water Flouridation***
b. Fluoride toothpaste.
c. Dental health education programs.
D. Individualized oral hygiene care.
604.597-Patient with extensive crown fracture with RCT and
discoloration indicated for post and core treatment the following is
the first step of treatment:
A-removal of gutta percha by a hot plugger
B-perform core build up
C-Remove any debris, caries or old fillings***
605.598-Worst type of bone for implants:
A- type I
B-type II
C-type III
D-type IV***
606.599-After performing pulpotomy with formocresol hyperemia of the
pulp indicates:
A-advanced degeneration of the pulp****
74

B-necrosis
C-there is still pulp tissue in the pulp chamber
607.600-During try in of PFM bridge restoration the patient said he feels
as if a seed is stuck between his teeth this indicates:
A- Tight proximal contacts***
B-over extended margins
C-under extended margins
608.601-Child underwent dental treatment for lower right lateral and
centrals the pts. Father called the next day informing you that there is
an ulcer on the lining mucosa on the right side of the lower lip
this is most likely due to:
A-Apthous ulcer
B-traumatic ulcer due to masticatory trauma***
C- herpes
609.602-schicks test is for :
A-TB
B-Diphtheria***
C-HIV
610.603- Disinfection of the impression by spraying includes:
A-the impression surface only
B- the impression surface and the bottom of the tray
C- the impression surface, bottom of the tray and the handle****
611.604- The concentration of household bleach is:
A-5.2%***
B-2%
C-8%
D-0.5%
612.605- patient suffering from cleft lip and palate, lower lip pits and
ankyloglossia
Is likely to have:
A-Treacher Collins syndrome
B-Van Der Woude syndrome***
C-Cleidocranial dysplasia
613.606-patient with good tissue support and normal structures with U
shaped palate wanted to construct a prosthesis that will be easy to
reline and maintain later, the choice of material used is:
A- metallic
B-metallic with acrylic
C-Acrylic***
614.607- Function of the indirect retainer in RPD is to :
A- prevent dislodgement along the path of insertion***
B-prevent movement towards the tissues
75

615.608- Patient with multiple purple bruises on his body, bleeding of


the gums and menorrhagia, the most probable diagnosis is:
A-hemophilia
B-thrombocytopenic purpura***
616.609- Patient on was on warfarin therapy and stopped what will he
most likely be of concern:
A-prolonged coagulation time
B-prolonged PT***
C-prolonged PTT
617.610- Pulp pathosis in primary dentition appears at:
A-apex
B-furcation area***
618.611-The following is used to fabricate implants :
A-Titanium****
B-nickel
C-palladium
619.612- patient with pain but cant localize which arch ,how can we
differentiate:
A- Anesthesia test***
B-cavity test
C-cold test
D- Electric pulp test
613-which part of the periodontal instrument should be parallel to the
long axis as much as possible:
A-blade
B-shank***
C-cutting edge
620.614-relation between the instrument and the tooth during operation
is called:
A-angulation
B-adaptation***
C-activation or smth..
621.615-The main purpose of metal try in in fixed full coverage
prosthesis is:
A- Check the fitting of the inner surface***
B-check occlusion
C-check proximal contacts
622.616-calculate the odds ratio:
Oral
Control
cancer
Smok
90
10
er
76

Non
smok
er

40

60

i.
ii.
iii.
iv.

6
2.35
3.43
13.5 ***
Odds ratio: 90 x60 / 10x40= 5400/400 =
13.5
Explanation:
In statistics, the odds ratio is one of three
main ways to quantify how strongly the
presence or absence of property A is
associated with the presence or absence of property B in a given
population.
623.617-Reversal lines indicate the end of:
A-Osteoclastic activity***
B-osteoblastic activity
C-fibroblastic activity
624.618-case about relining of Complete Denturesays something
about an old woman who had dentures for several years and they
beame loose but all is ok except retention what should be done???
625.619-Case about scleroderma (same as in files) patient with shiny
palms and forehead and resorption in the mandible BLA BLA BLA.
626.620-case about tissue conditioners use for inflamed ridge
627.621-Calcuim hydroxide is the best pulp capping material because?
a. a.It has the best seal over the pulp
b. b.it hase alkaline+less irritating to pulp
c. c.it induces reparative dentin formation***
628.622-The periodontuim comprise which of the following ?
a. a.Gingiva and PDL
b. b.Gingiva, PDL, alveolar bone
c. c.Gingiva,Pdl,alvealarbone and cementum****
d. d.Gingiva,PDL,alveolar bone,cementum And Enamel
629.623-Floss used to
a. a.Remove interproximal plaque***
b. b.remove over hangs
c. c.stimulate gingival circulation
630.624-Teeth with Rct and you want to use post ,Which post is the least
causetooth fracture?
a. a.Ready made post
b. b.Casted post
77

c. c.Fiber post***
d. d.Prefabricated post
631.625-Toosth had Truama resulted in hyperemia?
a. a.Pain increased with cold
b. b.reversible condition
c. c.pain of short duration
d. d.B and C
e. e.All***
632.626-PL complains from Pain in 45 which had gold onlay the pain
cold be due to?
a. a.Chemicals from cement
b. b.High thenrmal conductivity of gold ****
c. c.Related to periodontal ligament
d. d.Cracked tooth or fractured surface
633.627..Neonatal teeth eruption
a. At birth
b. 0-30 day***
c. After 6 month
634.628. 2.oculomotor nerve problem in? : T effects eye movement***
635.629.What the influence of xylitol: ?
a. A.it causes caries.
b. B.safe to the teeth.((reduce caries)) ***
c. C increase saliva.
d. D.decrease saliva.
N.b 3ry dentin: dentin present under irritation
636.630.Cracked tooth syndrome is best diagnosed by?
a. Radiograph.
b. Subjective symptoms and horizontal percussion.
***
c. Palpation and vertical percussion.
d. Pulp testing
637.631.Solitary bone cyst management:?
a. Anti-inflammatory and follow up.
b. Curettage and close. ***
c. Marsupialization and antibiotic
d. No active management
638.632.Gamma phase 2?
a. A.2% copper
b. B.4% copper
c. C.10 % copper
d. D.13 % copper***

78

639.633.Patient came to dentist after previous stressful procedure


complaining of burning & discomfort of his lip on examination u found
lesions on the palate, diagnosis is?
a. A.contact dermatitis allergy
b. B.aphthous ulcer
c. C.herpes simplex ( herpetic gingivostomatitis ) ***
640.634.An examination of the edentulous mouth of an aged pt. Who
has wore maxillary complete dentures for many years against six
mandibular teeth would probably show:
a.cystic degeneration of the foramina of the anterior palatine nerve.
B.loss of osseous structure in the anterior maxillary arch. ***
c.flabby ridge tissue in the posterior maxillary arch.
D.insufficient interocclusal distance
641.635.During 3/4 crown preparation on premolar, bur used to add
retentive grooves?
a. A.radial fissure.
b. B. Tapered fissure***
642.636.There is something like pearl
643.637.More flexible
a.k file
b.h file
c.broach
d. Reamer****
644.17.Cross section of file
reamer ?
645.638.What is the number
of pharyngeal "branchial"
arches:
a. 4.
B. 5.***
c. 6.
D. 7.
646.639.Slowest onset of action :
a-lidocaine
b-bupivicaine***
c-mepivicaine
d-prilocaine
647.-640.pulp polyp is:
a-hyperplastic lesion***
b-hypertrophy of the pulp
c- necrotic lesion
79

648.641..patient has thrombocytopenia :


a-prolonged bleeding time****
b-prolonged pt
c-prolonged ptt
649.

b.
c.
d.
e.
f.
g.
h.

642.after
class v gi
restoration
removal of a
thin flush of
gi is done
by:
a. Sca
ller
or

knife immediately.
Finishing stone immediately.
Scale or knife later.
Finishing stone later.
A + b.
A + d. ***
A+c
D+c

650.643.Treatment of internal resorption involves:


a. Complete exacerbation of the pulp to arrest the resorption process.
***
b. Enlarging the canal apical to the resorbed area for better access.
c. Utilizing a silver cone and sealer to fill the irregularities in the
resorbed area.
d. Filling the canal and defect with amalgam.
e. Sealing sodium hypochlorite in the canal to remove the
inflammatory tissue necrotic in the area of the resorption
651.644.Provisional restoration in anterior ceramic preparation
a.gi cement
b.polycarbonate***
c.amalgam
d.composite
652.645.Mechanical tooth brush invented in 1939***
80

653.646.A patient that wasnt anaesthetized well in his 1st visit, next
day he returns with a limited mouth opening ( trismus ) he must be
anaesthized, whats the technique to be used:
a) williams technique.
B) vazirani-akinosi technique *** (berchers technique)
654.647.To remove a
broken periodontal
instrument from the
gingival sulcus:
a) schwartz
periotriever. ***
655.648.Pt come t ur
clinic and he has
hyperventilation ?
656.649.Ttt of
anaphylactic shock ?

657.650.Small access cavity with not romval all pulp tissue and
discoloration the tooth
>> remaining pulp tissue***
658.651.10 years child presented with space between maxillary incisors
and have problem in
a. Options
b. Over jet ***
c. Anterior cross bite
I am not sure about this cause I dont remember the q and the
choices clearly.
659.652 12 year patent lost 63 and 64 what the treatment
81

660.Intrem rpd
661.Leave****
662.Fixed bridge
663.Rpd
664.653.How much time need to complete epithilization to occur
7-14 days is the right answer****
665.654.Defenetion of 3ry dentin:
A.irregular 2ry dentin
.scelerotic or
C.Reparative
A,c***
Lesion 2 cm of oral cancer, no metastasis, no spread to regional .655
lymph nodes
and asked for the right tnm classification
t0n0m0-1
t1n1m0-2
3-t2n1m0
4-t2n2m1
The right answer here is T1N0M0

82

.666

667.

656.Female pt with cd with angular chelities and wrinkles around her


mouth.
Low vertical dimention***
668.657.Enamel lamella
669.

670.658.Gutta percha contains mainly:


a. Gutta percha 20%. Zinc oxide 70%. ***
671.659.Which muscles cuase elevation of the mandible ?
Masseter , Temporalis and medial pteregoid****
672.660.Hemisection means:
a. surgical removal of the apical portion of the root. ( apicectomy ).
b. removal of one or more roots. ( amputation ).
c. the root and the crown are cut lengthwise. ***
d. none.
Mahmoud and Sobhies Exam :
673.661. A 45 years old female patient came to your clinic with an X-ray
the patient is suffering from anemia and without fracture of the
mandible with radulocency
83

a. What I remember from choices


b. Traumatic bone cyst***
c. Osteomylitis
674.662.9yrs child patient came to clinic there is a lot of talk couldnt
understand or remember anything about it although there was a
photo?
The answer is congenital abscense of upper central incisor from the
picture ****
675.663.patient with gingival bleeding what type of brushing we use in
this case ?
a. Modified Stillman technique***
676.664.best Pontic used in esthetic zone ?
a. Modifiend ridge lap****
677.665.young patient with missing single tooth with good gingival
condition (healthy gingiva) Have Myriland bridge we use it with
a. Broken abutment
b. Recurrent caries
c. Periodontitis
d. Staining****
678.666.what is the percentage of proximal caries ?
a. A 60%****
b. B 30%
c. C 50%
d. D 100%
679.667.The main use for electric Pulp testing ?
a. A Nueral vitality****
b. B Vascularity
c. C periodontal ligament
d. D Cementum
680.668.Patient with viral hepatitis type C he got injured in the clinic
what to do ?^^^^Its written in files I couldnt remember which file
a. A take him to the doctor responsible for communicable diseases
****
b. B apply pressure to stop bleeding
c. squeezing of wound
d. D put plaster for injury
681.669.9 years old patient there is a history of trauma in the upper
central incisor and there is a bluish discoloration in the teeth. Calcium
hydroxide pulpotomy is done 1 year ago What to do ?
A wait till the patient is 18 years old
B wait for complete root formation****
C wait until we construct abridge
84

We have to wait for apex closure since the trauma was 1 year ago the
patients age is 8
682.670.amalgam as a retrograde filling
A free zink***
B added copper
A retrospective study was done to compare the success rates of teeth
with three different root end filling materials. The materials studied
were SuperEBA, IRM, and zinc-free high-copper spherical amalgam.
Radiographs of 488 cases from two geographically distinct offices
were used, with the recall period ranging from a minimum of 6
months to a maximum of 10 yr. The cases in each office were
examined independently using the same criteria. The results revealed
that both SuperEBA and IRM demonstrated statistically significant
improvements in success rates when compared with amalgam. The
success rates were 75% for amalgam, 91% for IRM, and 95% for
SuperEBA. The difference between IRM and SuperEBA was not
statistically significant.
683.671.patient comes to clinic with porcelain fused metal bridge at
lower 6 The patient feel some tightness in the Pontic ?
a. A improper preparation of abutment
b. B insufficient relief of the fitting surface***
c. C occlusal wear
d. D something about sticking dont remember exactly
684.672.after extraction of a tooth you found pearl like prominence in
the root?
Enamel pearl****
685.673.a case of malignancy come from the pathologist 1.5 cm
malignant tumor whats the treatment here ?
a. A incisional biopsy
b. B chemical therapy
c. C Excisional Biopsy***
d. D curettage***
686.674.a child came to your clinic to have a treatment after you finish
treatment he came after a while with a trauma in his face with a
swollen lip and gingiva with healthy teeth whats the treatment here?
The answer here what I remember is antibiotic and follow up***
687.675.what is the function of posterior palatal plan for children?
a. A prevent crowding
b. B treatment of deep bite
c. C treatment of crossbite****
688.676.pt has something like injury and bleeding what is the type
ofpaste u will put it in the tooth to assist heal and the type of local
anathesisia u will use to assist stop bleeding
85

a. ****A.50%bubvicaine+1:200000epinephrine+caoh paste
b. b.2%lidocaine+1:00000 epinephrine+GI
689.677.aquestion about rieters syndrome wich affects the eye

690.
691.678.question about hyperparathyroidism

Good tips

692.679.keratosis on the scalp Gorlins syndrome something about it

Gorlin syndrome is a rare condition. It is also sometimes called


naevoid basal cell carcinomasyndrome (NBCCS). You may also see it
written as Gorlin's Syndrome
Most people with Gorlin syndrome also develop noncancerous
(benign) tumors of the jaw, called keratocystic odontogenic tumors.
These cancers occur most often on the face, chest, and back.
693.680.Albright syndrome
86

694.681.question about percentage 97% something like that its


mentioned in files but dont know exactly which file
695.682.a disease which is slowly growing and affecting parotid gland in
afemale patient?
Pleomorphic adenoma***
696.683. question about adenoid cystic carcinoma
697.684.question about population and country Risk factor
698.685.which one of these techniques is the easiest ?
a. A pulpotomy
b. B pulpectomy
c. C partial pulpectomy
d. D capping****
699.686.question about abfraction?
Abfraction is :occlusal stress leading to excessive tensile forces which
cause damage to enamel at the cervical areas of the teeth.
700.690.question about facebow. uses
701.691.question removable the labial surface is clear while the lingual
surface is not clear ?
a. a.maximum cavarouge
b. b correct contouring
702.692.the best material used for intercanal pulpectomy?
a. A Zoe****
b. B Calcium hydroxide
703.693.you want do apecictomy in the lower canine what is the type of
anesthesia?
a. A Inferior alveolar nerve block with long buccal nerve block
b. B inferior alveolar nerve with.infiltration of lingual surface
c. C Mental nerve block***
704.695.intrapulpal diameter
doctor sheriffs file
705.696.question about the plaque color complex it was a long case
sorry I couldnt remember anything from it (what I remember) is the
patien was in clinic and you treated one quadrent from plaque. The
color complex of it changes in color . what is the color complex?
The answer was purple I think
706.697. Child with comminication problem diffculties in verbal and
emotional intercation with people have involuntery motion
1.down syndrom
2.autism
3.cerbral pulsy***
707.698. Odontogenic tumer
1.arise from teeth tissue****
87

2.rarely malignant
3.radiolucent in x-ray
708.699. Antirust agent used for dental burs
1.sodium nitrate***
2.silver nitrate
709.700. Till when you keep file of patient in dental office
#5 years from last visit
#7 years from last visit***
710.701. Pedo patient with history of malaise and fever have grey
yellow vesicles in hard palate and in
gingiva
Diagnosis
1.rubella
2.herps infection****
3.apthous ulcers
711.702. Patient with hypophosphatemia what you will In his blood
test?
712.703. Patient with gun shot in mandible bone it did not say which
area bone the fracture is commnuated what is technique of bone
redction
4

713.kenen technique
714.704. Pedo patient during eruption of first permnant
molar there is resorbtion of primery second molar root what is your
mangement?
715.705. 12 years old female, has badly decayed 36, 46 nonrestorableafter extraction you do:
A. partial denture
b. FPD
C. leave without any treatment
D. interim partial denture****
716.Careful in exam they might change numbers of teeth and age its
very important to notice that
717.706. Question about Reiters Syndrome?

718.
719.707.question about cherubism.

88

720.
721.708.question about Gardners syndrome?

722.
723.709.question about pleomorphic adenoma?

724.
725.710.question about smoothening of CsA
726.
727.711.question about intracanal pressure

89

728.
729.712.The difference in growth between boys and girls ?present in
filescant remember where
730.The answer is girls are 2 years ahead of boys
731.713.Non resorbable material ?
732.4-6
733.714. Growth of median border of lip?
734.A Fronto nasal****
735.B First pharyngeal arch
736.C Maxillary process
737.D Mandibular process
738.715.-calculate the odds ratio:
739.
740.O
741.Co
ral
ntrol
ca
nc
er
742.S
743.9
744.10
mok
0
er
745.No
746.4
747.60
n
0
smo
ker

90

748.

i.
ii.
iii.
iv.

6
2.35
3.43
13.5 ***
Odds ratio: 90 x60 / 10x40= 5400/400 = 13.5
749.Explanation:
750.In statistics, the odds ratio is one of three main ways to quantify
how strongly the presence or absence of property A is associated with
the presence or absence of property B in a given population.
751.716.gingival changes in smokers shows
752.Less than non smokers
753.717.picture showing a fracture of a denture from anterior to
posterior from the mid line with resorption of ridge and pain for 15
days?
754.A the frenum or post dam***
755.B improper adjustment of oclussal forces
756.C and D the choices are from another planet
757.A is the nearest answer but the best answer is Ill fitting denture with
lack of relief of median palatine raphea thats the answer****
758.718.a picture of X-ray showing lower primery molar with exposure
of mesial pulp horn due to caries while the distal surface is intact?
759.A Pulpotomy****
760.B pulpectomy
761.C DPC
762.D IDPC
763.719.Agun shot of face which type of alveoloplasty procedure we
use?
764.A Traditional alveoloplasty
91

765.B Clarks alveoloplasy***


766.C some scientests which I dont know
767.720.The best way of management of thumb sucking?
768.A Stop it before eruption of upper anteriors
769.B Stop it before eruption of lower molar
770.C encourage to stop this habit from relatives
771.721.Preventive measure is
772.A change in behavouire****
773.B Treatment of problem with encouraging
774.C .
775.722.4 yrs old child with first time dental treatment in your clinic
what effects the child behavouire is
776.A the behavoure of the child before the first visit
777.B behavouire of parents in house
778.C the behavoure of sibling to dentist****
779.D the behavoure of the neighbours
780.723.patient under orthodontic treatment he has pulp exposure in
single tooth when is the right timing for obturation?
781.A before ortho
782.B after ortho
783.C during ortho****
784.D ..
785.724.base under composite?
786.A Ca(oh)2
787.B zincphosphate
788.C calcium hydrate
789.D zinc oxide eugenol
790.The answer here all except D
791.725.Peire abutment is ?
792.A Distaly missing two teeth
793.B Mesialy missing too teeth
794.C missing one tooth mesial and one tooth distal***
795.726.question about arterial blood pressure I dont remember exactly
but ithink its about systolic pressure or diastolic:
796.A 35 40 m/m
797.B 10
15
798.C ..
799.D ..
800.727.question about the S.S crown which was the same size of crown
801.But rotate:
802.A trim the s.s crown
803.B select smaller size****
92

804.C select the smaller size


805.D mix thicker layer of cement
806.728.Child has Hiv disease what is most likely to have ?
807.A herpetic gingiva stomatitis****
808.B Xerostomia
809.C ..
810.D
811.729.patienst with a bluish gray discoloration of all teeth with
enamel chipping and havent equal bone on the roots. One root is
taller than the other?
812.A amelogenisis
813.B dentinogenisis****
814.C dentin dysplasia
815.730.space between maxillary and mandibular teeth anteriorly what
is it called?
816.A Free zone
817.B lee way space
818.C free way space****
819.731. question about cyst but I dont remember exactly but its
written in files
820.732.q question about gun shot in the mandible what is the
instrunment used for treatment here ?
821.733. tunnel preparation is used?
822.A proximal***
823.B Occlusal
824.C ..
825.D ..
826.734.question about a case in which patient came to your clinic and
wants to measure something called gentle Tempreture ?
827.A the tempreture increased and then later decreased
828.B decreased and then later increased
829.C deacreased and remains
830.D stable then decreased
831.735.what is the main cause of discoloration of upper central incisor?
832.A Tetracyclin
833.B recurrent caries
834.C gutta percha from endodontic treatment***
835.736. According to the curve of (postnatal growth) the system which
reach its maturity the peak first is ?
836.Muscular
837.Genital
838.Neural***
839.Lymphatic
93

840.
841.737.a village with population of 100 , 35 were diseased per year, in
7 years study what is the prevelnce of the disease ?
842.a-35
843.b-50
844.c-25
845.d-245***
846.738. current for un-simulated nerve:Nerve cells have a resting
membrane potential of around -70 mV, close to the Nernst potential
for K+ (-90 mY)
847.a.+50
848.b.-50 -70***
849.c-+80 +90
850.d.-80 -90
851.739. 2.step deformity in mandibular fractures is caused by :
A) upward pull of temporalis and masseter ***
B) lateral pull of medial and lateral pterygoid.
C) downard pull of geniohyoid and infrahyoid and digastric.
D) due to mylohyoid muscle
A is the best and correct.for sure . But take care if he said in the
options A and B try to choose it . If he didnt choose A only.
852.740.what is the space needed in mm. in primery teeth so
crowdining wont happen in permenant successor ?
94

853.741 a patient he is a teacher. Comes to your clinic .he got no


problems in pronounciation except for F and V letters what is the
problem here ?
854.A too short upper lip
855.B too long upper lip
856.C low vertical dimention
857.D upper anterior teeth flair upward****
858.742.question about galvanic action between two restorations upper
gold and lower metallic there is a pain on biting from ?
859.A occlusal disharmony
860.B thermal conductivity
861.743.where to place the finish line in case of gingival recession?
862.We place the margin or finish line at the gingiva
863.N.B For the prevention of recession we place the margin at the
gingival 1/3 of the crown.
864.The best type of margin is placing the margin above the gingiva
while the most esthetic margin is subgingivaly.
865.744.what is the best instrument used in making of finish line >?
866.A Tapered stone ***
867.B End cutting diamond bur
868.C
869.745. what is the type of finish line in PFM ?
870.A Shoulder
871.B Chamfer
872.C Shoulder with bevel***
873.746.Patient with 15 yrs old age what type of cement to place under
esthetic restoration class III, beneath it
874.A Zoe soft creamy
875.B Zoe with good isolation and set
876.C Zoe combination with some cement
877.D. light glass inomer***
878.747. cbc test
a .Wbc.RBC. platelet. HB.HCt****
b. WBC.RBC . Platelet.HB. HCT. CA
c. WBC. RBC. platetel. HB. HCT.CA.CL
879.748. Chin cup used for ttt of
A-class 3 with mandibular protrusion****
B-class 3 with long lower face
C-class 3 with maxillae deficincy
D-pseudo class 3
880.749.ttt of solitary bone cyst in premolar area
a-antibiotic
b-marthoplization and closure
95

c-curettage and closure ***


d-........
881.750. Child patient came to your clinic with his nanny .. You want to
take dental history what will you do
A-take it from the child
B-from his nanny
C-call his parent ***
D-......
882.751. photo of upper and lower record block extraoral from posterior
view with upper and lower denture base contact each other
posteriorly
then ask about what will happen if we neglect this contact
a-unbalanced occlusion
b-deviation between centric relation and centric occlusion***
c-premature contact between functional cusps
d-..........
883.752. uncontrolled diabetic patient with swellin at lower molar area
due to badly decayed tooth ...patient is sensitive to penicillen what to
do
a-amoxcillen then rct
b-rct then amoxicillen
c-clarimycin then rct
d-rct then clarimycin***
884.753.study reveals that incidence of disease is 350 from 10000 for 7
years what is the prevalence of the disease
a-350
b-25000
c-35
d-...................
885.The answer is 350*7 if he asks about the prevalence in 7
years=2450
886.754. patient with sub condylar fracture withh class 3 at one side
and class 2 at another side... you can return mandible to normal class
1 with your hand ...ttt
a-close reduction***
b-open
c-wait

96

887.

888.
889.755. probe used to diagnose furcation involvement
a-who probe
b-nabers probe****
c-......
97

890.756. coronal part of root cementum


a-intrincic acellular cementum
b-extrinsic acellular cementum***
c-cellular cementum
d-.....
891.757. determining of working length
a-good periapical bisecting is sufficient***
b-pass the file to periapical area to determine
c-use precurved file
d-.......
892.758. patient with multiple posterior teeth restoration, gingival
recession,bad oral hygine came to restore badly decayed lower
molar ... ttt plan
a- composite
b-complex amalgym****
c-extraction and bridge
d-glass ionemer***
893.759. 9 years old child you note white lines which are more
concentric in cervical than incisal ... identify
a-enamel lamella
b-von ebner straie
c-enamel tuft****
d-wikham straie
894.760. it is normally to remove frome 1 to 3 mm of affected coronal
pulp..name this
a-direct pulp ttt
b-indirect
c-pulpotomy
d-partial pulptomy***
895.761. 6 years old child came to your clinic for 6 months routine
dental examination , in the previus visit he had x-ray invistigation and
there is no caries what will you do this visit
A-take 2 bitewing for the left and right
B-take 2bitewing and panoramic
C-take 2bitewing and occlusal film
D-no need for x-ray***
896.762 about diagnosis of disk perforation
a-arthrography****
b-mri
c-ct
d-.....
897.763.after orthodontic ttt you note white discoloration in ant teeth
how to manage
98

a-micro abrasion****
b-pumice
c-hydrogen peroxide
898.
764. femal patient came to your clinic with white patch in an area
under complete denture after reliefing dentur at this area white patch
disappear but appear at another ..cause
a-un even distribution of occlusal forces***
b-high vertical
c-premature contact
d-.........
899.765. Q about anterio liability
a-posterior teeth-permanent successor
b-4 incisor permanent successors
c-primary anterior teeth permanent successor****
d-..............
900.The definition of incisal liability: Mesiodistal width of 4 permenent
and primary their diffrence is the incisal liability
901.766.Patient wants to do Fpd and one of the abutments show
stressed pulp what is the stressed pulp ?
902.a.tooth need Rct
903.b.tooth have restoration
904.c.tooth have force on it
Stressed pulp is The stressed pulp condition is a clinical concept that
describes pulps that have received repeated previous injury and
.survived with diminished responses and lessened repair potentials

.905

Before performing restorative dentistry the dentist should conduct a

.906

.comprehensive pulpal health evaluation on teeth to be restored

Gutta percha transfer into .767


?alpha phase in which temperature
*** .A) 42_49 c
B) 50 - 58

99

.907

909..
910.768. Histopathological,dentigerous cyst lining epithilium may be :
1_stratifi. Squamous epith.
2_reduced enamel epith.
3_cuboidal shape
4_all of the above.****
The Answer here (D)all of the above

911.

100

912.
913.769. Avulsed tooth, u told parents that they have to come to the
office for might reimlpantation,?what would you told them to put the
101

Tooth in??
Water
Salin
Cold milk***
Milk at room temperature

914.
915.770. Conventional class || cavity occluso- mesial , there's no
gingival seat due to caries, how dentist creat it?
A)Make gingival seat with amalgam
B)Make it with composite
C)make it with glass ionomer****
D)extend the axial root on root surface
916.771. Q.disinfection of gutta percha??
772.liability of anterior teeth minimum space ....not defect upp or
lower(3-4-7-8
917.
.773.the thickness of the cusp builded by amalgam(2-2.5-10)
918.
774.first time pedo checked by orthodontist
775 about hemophilia one for diagnosis and one for type and
factor...pt after ex have sever bleeding and factor 9 less than
10%.normal bleeding time?
919.A Heamophilia A
920.B Heamophilia B***
921.C.Heamophilia C
922.N.B Hemophilia B, also called factor IX (FIX) deficiency or Christmas
disease, is a genetic disorder caused by missing or defective factor IX,
a clotting protein. Although it is passed down from parents to
children, about 1/3 of cases are caused by a spontaneous mutation, a
change in a gene.

102

923.Severity
924.(percentage breakdown of overall hemophilia population by
severity)
925.Severe (factor levels less than 1%) represent approximately 60% of
cases
926.Moderate (factor levels of 1-5%) represent approximately 15% of
cases
927.Mild (factor levels of 6%-30%) represent approximately 25% of
cases
928.776.the IGA from mother to fetus which type of immunity? Typically
As file.
777.picture of denture wth mid line fracture till post.palat.seal what is
the cause.......typical as file
929.
778..picture pt with extracted 4.6.7.8 .&5 has mobility grad 2 what is
the design of partial denture?
930.(extract 5 and make partial-form over denture with 5 is abutment...)
779.mongolism is or Downs syndrome?
931.(disomy 21.or trisomy) ***
780.Horners SYNDROM??
932.EFFECTED NERVE IS

933.
781.pt whith fracture in angle of man...xray is
934.A supmentovertex
935.B -lateralobliqe
936.C -postroanterior)
103

.......

937.
782.tip of tongue drain in .......sub lg same as (file)
938.
783.tongue thrust due to
939.(large tongue-intenisty swallowing)
940.
784.the blade to the handle is (parallel-perpendic to long axis of
handle.) same as Files
941.
785 .is the sequance of file (21-25-31)same as file
942.
786.alginate impres.delayed 15 minute then pour cause chalky
appear of cast-why???? Dryness of impression..file
943.
787.neonatal teeth Q IN FILE .
944...
788.lesion in fetus with ant.premaxilla...
945.ans....cong.epilus of newborn.
946.789.syndrome and Crouzn diz?

947.
948.
790.instruments T.burnisher and curved hemostate was repeated
949.791 about osteogensis with strange choices ?
950.
792. about fibroblast ingrowth also new and strange
951.
793 about clasp in the file akers clasp
952.
794 about Ala tragus line also new the choices
104

953.
795.q about 3 cases of young permenant teeth with trauma
954.pulptomy/apexogensis/apexofication (study them well)
955.
796. about intrinsic pathway problems and PT PTT sth like this
(elmohom lazm t3rfo ay factors fe intrensic and extrenic pathways)

956.
797. about water flourdation but not with PPM unit i gues by mg/kg ?
798 about CMCP this one in the files ?
957.
799.q about open faced partial veneer with strange choices !
958.
800.q about MTA msh mtzkra el choices exactlly but its superior to
other materials
959.
801 about ttt also in files fo TB pt written in files
960.
802 about high cooper amalgam
961.
803 stephan curve with questions on it

105

962.
963.Acidogenic bacteria in dental plaque rapidly metabolize fermentable
carbohydrates producing acidic end products. In the mouth, these
changes over time in response to a challenge (usually a cariogenic
food) are known as Stephan responses or Stephan curves.8 The pH of
dental plaque under resting conditions (i.e., when no food or drink has
been consumed), is fairly constant. Differences do exist, however,
between individuals and in different sites within an individual.
964.The response after exposure of dental plaque to a fermentable
carbohydrate is that pH decreases rapidly, reaching a minimum in
approximately 5 to 20 minutes. This is followed by a gradual recovery
to its starting value, usually over 30 to 60 minutes, although this can
be longer in some individuals.
965.
804.q about aphtus and herpes
966.
805.q about cementation of inlay ?
967.
806.q about color of gingiva (kertinization/meantoin/vasculirty/....) o
el choices kant combination mn hay el choices
968.
807.q about band and loop appliance main use in ortho
969.808.1-3 mm exposure into the pulp Partial pulpotomy
970.809.pt with blah blah blah nd shiny hands
Scleroderma
971.810.pt with multiple osteomas nd mutiple cysts Gardner's
972.811.pt on radiograph has sunshine appearance Osteosarcoma
106

973.812. _bilateral loss of upper d's Nance


974.813.HBV how to deal with it (they got all the options that U'v posted
b4, with the same organization but they gave me other combinations)
1,3,4 2,3,4 1,4 1,3 written in files
975.814.remodeling of periodontal ligament is by?
976.A Osteoblast
977.B Osteoclast
978.C Fibroblas
979.815 _pt with Condylar fracture, what graft should b done
980.Chostochondral
981.816 _trauma is more common with which of malocclusion
982.A Cl 1
983.B Cl 2***
984.C Cl 3
985.817_pt with severe bone resorbed, what techniques should b used
for impr Mucocompressive
986.Stretch O alvagy some other choices cant remember

987.818._pseudo class 3 is treated with


988.Laibial bumper O Bardo ma mutzakir albaggy
989.819._pt took antidepressants injection , the punch side became red,
y?
990.Something in their contents fiha phynol aomthing kida bas mush
fakir bardo
991.820._pt with enamel discoloration nd he u made cavity with minimal
enamel prepare what does this procedure called,
992.A Pits nd fissures
993.B Preventive****
994.C Conservative
995.821.MAF MASTER APICAL FILE
996.First file binding to apex in working length
997.File used for debritment of canal
998.Two other options o kolahom kano sa7 bas Ana ma mutzakirom wlai
999.822 _ttt of mucosele
1000. Excision Incision
1001. 823_Thermomechanical teq
1002. Macspadden
1003. 824.Patient needs complete denture how many implants will you
use ?
1004. 825._pt asked u to change his c.d nd u checked it nd it was good
nd the pt saied he had three b4 this one, what is the type of the pt
107

1005. 826.cells that migrates to side of infection?


1006. PMNs Leukocyte
1007. 827._contraindications of gingivectomy ?
1008. Perio abcs
1009. 828_2 questions about he anesthesia lkn bardo ma mutzakerom
Fihom wa7id 3n how many carboles for a 4 Yeats child pt
1010. 829._pt with trauma before 8mnths ago came with no
symptoms,what will u do
1011. A Pulpotomy with caoh
1012. B Complete debritment with apixification
1013. Treatment depends on the age of the child so we know if the
tooth is perm or Dec or root is completely formed or open apex
1014. 830.child 5 years old took antibiotics when he was 8 mnths
ago,wht tooth will b discolored
1015. 831.thyroid gland origin
1016. 4th.pharangeal arch Parafollicular part
1017. 832.space mantainer ....factor important in select mantainer )
1018. A chronological of th pt
1019. B -early eruption of ant permanenr
1020. C -late eruption of perm.molars

1021.

108

1022.
1023. 833.crowding of anterior
teeth in age 18 y due to
(Impaction of 8-ortho.relapse-physiological pupirity changperiodontitis)
1024. 834.pt has large caries pain with cold only which has spontinous
episods.ph has no pain in percussion or palpation no x ray changes
.what is the pulp condition???
1025. A -reversable
b irreversable
c we need more information cant detect pulp condition
1026. 835.the root of internal resorption has
1027. A -reversaple pulpitis
b irreversable pulpitis
c necrotic
d normal
1028. 836.crown..Q.pt with upper central incisor with highly stained
have mesial caries whith chip of part of incisal edge crown best to be
a all ceramic
b .ceramo metallic
c co.cr metal
1029. 837..pulpitis from composite is mainly due
a .monomer
b polymer sorry cant remmember 2 choise
1030. 838.the most efective elevat &protrude the mandible
a masseter
109

b .temporalis
c lateral tregoid
1031. 839.on pontic that is convex from all side?
1032.
840.about pedo after extraction he has massive bleeding and
osteomylitis in the next day.the blood test has anemiathrompocytopenia -wbcs 10.000.desease is
1033. 1-leukemic leuckemia
2.Agranulocytosis...2 other not remmember away from cases
1034. 841.Q.Denture .pt after 1 day complete denture come with psin
under denture base.there are redness spots under denture base .pt is
on anti histamenic cause due
1.anti.hestamine adverse rx of drug
2.hyprsenstivity of denture material*** or (Contact stomatitis )
3.sore moth .
1035. 842 -Child 6 year have abnormal enamel dentin and pulp in A
quadrant you diagnosis is:
a. hypoplasia
B. regional odontplasia****
c. Detogensisimperficta
d. Amelogensisimperficta
1036. 843----please write this .pt has case need endo
.post.core.restoration..whic facor is most import in this restoration
1.type of restoration
2.diameter of post
3.configuration on post surface
4.produce ferrule
844.the lmportant of all perioflap
1.remove granulation tissue
2.axcess for diseased root
845..tooth most likly have talon cusp and densEvagenation
1.upper 1&2
2.lower 1&
3.upper 2
Inferior alveolar nerve is branch from .846 .1037
a Mandibular nerve untrunked branch (main one)
b. Anterior division of mandibular nerve
***c Posterior division of mandibular nerve

110

Patient has pain of short duration with hot and cold .847

.1038

1039. Irreversible pulpitis


Necrotic pulp
Dentin hypersensitivity***
Symtomatic apical periodontitis
Patient come to ur clinic complain from sharp pain like electricity .848
on tooth 46 which had amalgam restoration just yesterday,, you find
that he had gold crown on 16 opposing to that tooth so it's galvanic
?shock, what will u do
Change restoration 1
***Put varnish 2
Put separation medium 3
Extract this tooth 4
wait

.1040

Child had carious. Lower molar with swelling and complain from .849
mild pain and on radiograph show mild extension on this area with
scattered radioopaque lines
a. Paget' disease
b. Fibrous dysplasia
***c. Garre's osteomylities

.1041

d. Osteosarcoma

.1042

1043.

111

1044. 850. which of the following events occurs during dentinogenesis:


A. Odontoblasts become long cuboidal. ***
B. The matrix and proteoglycans maturate with collagen fibers.

1045. Their shape change men ovoid > columonour and their nuclei
become basal in orientation
And they increase in lenght but the width bykon the sameeee
1046. 851. 7Y old pt with missing 2 c in both sides .what mainly the
reason
1.Congenital missing***
2.Early exfoliated
3.dentist extraction
4.Trauma
1047. 852. After truma in what stage Ankelosis for decidus tooth
occure:
1) formative
2) eruptive
3) rupeture
1048. 4
Picture: for lower 4 anterior teeth .853 .1049
Kandy class 1 the anterior has moderate resesion and degree of
: movement whats the treatment
extraction and make complet denture (1
***over denture after preparation teeth (2
swing lock partial denture (3
.... (4


1050. After truma in what stage Ankelosis for decidus tooth occure:
1) formative
2) eruptive
3) rupeture
4) ....
1051. 855. The test to determain pulp heal:
1) electrical
2) thermal***
3) percussion
112

1052. 856. Tight lower lip desplacment denture:


1) ...
2) upward
3) backword
4)...
1053. 857.Access opening in lower first molar is:
1) rhombud
2) rounded triangle base buccaly
3) rounded triangle base destaly
4) ....
1054. 858. The diffrent between sympathetic apical and sympathetic
( peri apical) :
1) entrocuccos fecalis microb
2) diffrent type of miobacteria
3) .....
1055. 859.Why we use caoh between visits in RCT:
1- antibacterial.****
2- formation hard tissue.
3- primary seal.
4 - resorption pathology.
1056. 860. During scalig and root planing ther is atachment loss whate
is the minimum depth of pocket for attachment loss to occur:
1)1,9mm
2) 2,9mm
3) 3,9mm***
4) 4,9mm
1057. 861. 1534. Child 3 years old with congenital heart disease and
has deep caries with diffuse abscess and he transfered to hospital for
special management. What they will give the child before start:
A- endocarditis prophilaxis.
B- intravenous antibiotic.
1058. 862. which part of periodental instrument is Parallel to tooth ...
1059. 1 handl
1060. 2shank
1061. 3 cutting edg
1062. 4 blade
1063. 863.Spedding principle:
i. Used for selection of stainless steel crowns***
b. Used for selection of restorations
c. Used for selection of shade
d. Used for selection of sealant
1064. 864 Fractured tooth to alveolar crest, what's the best way to
produce ferrule effect:
113

A) Restore with amalgam core sub-gingivally.


b) Crown lengthening.
c) Extrusion with orthodontics.***

1065. 865.

? A question about Garres Sclerosing osteomyelitis .866

.1066

.1067
Nerve give secretomoter to parotid gland .867
A: facial
B: trigaminal
C: maxillary
... :D
1068. The right answer is Glossopharengeal

114

1069.
115

:The incisal edge preparation in porcalin fuse to metal .868


A: 1.5mm
B; 2 mm
C: 3mm
D: 1 mm

.869

.1070

.1071

? Distance between pt and cephalometric device feet -

.1072

****a-5 feet
b-6 feet
c-8 feet
d-Other
1073. 871. Innervation of the palatal gingiva of the anterior maxillary
teeth
A: nasopalatine****
B: greater palatine
C: glossoparyngeal
D.....

1074.
1075. 872.what type of bacteria which not present newly born child ?
1076. The answer is Strepto coccus mutans***
1077. 873. All are single bone in the skull EXCEPT:
a. Lacrimal. ***
b. Occipital.
c. Sphenoid.
116

d. Parietal.
1078. 874. Mastoid process is a part of:
a. Temporal bone. ***
b. Parietal bone.
c. Occipital bone.
1079. 875 To remove a broken periodontal instrument from the gingival
sulcus:
a) Schwartz Periotriever. ***

1080. 876.Blade of periodontal instrument should be:


A- Perpindicular to long axis.
B- Parallel to long axis.
C- Perpendicular to shank.
1081. 877. Gardener : multiple sebaceous glands at the back of the
neck and palms , osteomas in mandible , supernumery teeth and
impacted teeth.
1082. 878. Allis forceps : to hold tissues perior to excision
1083. Addison forceps : to hold tissues for suturing
1084. 879. 634.An examination of the edentulous mouth of an aged pt.
Who has wore maxillary complete dentures for many years against six
mandibular teeth would probably show:
a.cystic degeneration of the foramina of the anterior palatine nerve.
B.loss of osseous structure in the anterior maxillary arch. ***
c.flabby ridge tissue in the posterior maxillary arch.
D.insufficient interocclusal distance
1085. 880.management of avulsed tooth ?
1086. Apply a flexible splint for up to 2 weeks.
117

1087. 881.when you anesthetize upper 7 for extraction the patient


become pale and sweating in the side of anesthesia?
1088. 882.The diazepam Dose is ?
1089. 883.patient with asthma whats the sort of anesthesia you give?
1090. Ephedrine 1/1000
1091. 884.how Autoclave works ?
1092. Heating Steam
1093. 885.in amalgam functional cusp width ?
1094. 2ml***
1095. 886.biochemical properties for success Implant:
- coronal anti rotational
-tytinum alloy *
1096. 887.Cuase of porcelain brittleness :
-high ductility
-poor compressive strength
-poor tensile strength
1097. 888.Pt has fibroblast mutation and defect in chromosome 10
there's delayed closure of bonesutures also there's aye problem
(exoph ) the diagnosis :
-Albright syndrome
-cleidocranial dysto
-down syndrome
-Cauzon syndrome
1098. 889.Best ttt of intruded primary teeth :
-leave it to re-erupt
-surgical extrusion with stiplizer
-surgical extrusion without stiplizer*
-orthodontic extrusion with stiplizer
-orthodontic extrusion without stiplizer
1099.
890.Diabetic pt with loss of his lower 6 and 7 want to replace his
missing teeth . Best ttt :
-RPD with metal frame
-RPD with acrylic frame*
118

-mixed RPD metal and acrylic


-FPD
1100.
891.Pier abutment is :
-holds on tooth
-anterior to Edent area
-anterior and posterior Edent area to it***
1101.
892. 21 years old pt came to ur office for routine check up without
any complain when u take x-ray you found a R.L area around upper
central and lateral .. no swelling no pain the diagnosis is :
-radicular cyst
-osteofibroma*
-ameloblastoma
-granuloma
893.10 years old pt came with bilateral lower face swelling ....... pain
less ..... diagnosis :
-chirbisim*
1102. 894..NaCl used in RCT to :
-disinfect the canal
-softening the canal****
-kills microorganisms*
Nacl is normal saline . if its NaoCl then the answer is Cand A
1103. 895.Length of files and remears :
-20,24,29
-21,25,31*
-21,25,29
1104. 896.Pt with renal failure will discover :
-candida
-hairy tongue
-hairy leuokoplakia
-hyperparathyrodsm***
1105.
897.Pt cames to ur office want to chabge5 his old denture... on history
he told u that he had this denture 5 years ago .... With bad oral
hygiene ..u found a multilesions on his oral mucosa ttt :
-pharmacological
-alveoplasty
-vestibuloplasty
-recovary ttt *
119

1106.
898.Maximum time for avulsed tooth to be re-implanted is :
-1hr
-24 hrs *
-1 week
-8 to 9 hrs
1107. 899.Hard teeth to be anaesthesied :
-upper molars
-lower molars *
-upper premolars
1108.
900.Oral surgeon decide to give extraoral nerve block in pt with sever
trismus .. he inters the needle from outside after he reach the lateral
ptrygoid plate he must go :
- inferiorly and laterally
- inferiorly and medialy
- inferiorly and posteriorly*
- inferiorly and anteriorly
1109.
901.Porcelain margins used in :
-esthetic purpose***
- irregular finishing line *
-supra gingival.....
- ....
902.Cleft Lip and palate seen associated with :
-strickler syndrome
-Apertz syndrome
-down syndrome
-all *
1110.
903.Pseudo class regarding to anatomy :
-soft tissue over growth
-vertical growth of ramus
-........
1111. If there is non of the above choose it
904.Oculomotor nerve problem will cause :
-superior constructor m
- downward and lateral movement of eye***
-construct eye pupil
1112.
905.65 y old pt come to make complete denture in radiographic
examination there's sever bone resorption the crest of bone close to
120

mandibular canal best ttt :


- implant supports acrylic
- implant as over denture
- ......
-..
1113. 906.Child swallowed 10 mg fluoride ttt :
-milk followed by water
-milk***
-refer to hospital

1114.
1115. 907.32 y old femal came to the dental office with moderate pain
on upper central and lateral that treatred endodontically one years
ago ..the teeth are tender to percussion and the dentist shows
periradicular abscess at the same region and decided to redo RCT the
dentist must :
- leave the canal opened
-retrograde filling***
- gives antibiotics befor start
-..........
1116.
908.Pedo pt cames with sever calculus and inflamed gingiva with
pocket more than 3 .u will do scaling using :
-U.S
-ultrasonics
-peziuosonic (am not sure about name )
-manual *
121

1117.
909.Femal pt with rct tooth with mesial and Distal caries ttt :
- meso-occluso-distal only
-meso-occluso-distal inlay
- full gold crown***
1118.
910.Pt came complain of sharp pain with cold due to fracture of cusp
of the tooth confined to enamel ..the pulp status is :
- normal Uninflamed***
- reversible
- Irreversible
- pulps has A fibers only
1119.
911.Preventive community program :
-change behavior***
-prevent disease occurance
- ttt and reinforcement
- prevent recurrence
1120. 912.Enamel rods direction:
-perpendicular to outer surface
-perpendicular to DEJ***
-toward the dentine
-parallel to dentinal tubules
1121.
913.in pits and fissures sealant the etching process for :
-make surface bright
-cause roughness
-conditioning ***
-remove dark color
1122.
914.Unmodified class 2 Kennedy with last tooth lower 5 with undercut
mesialy best clasp :
-wrought with round end***
-wrought with half round. ...
-smooth with round end
1123.
915.All are correct regarding to pagetz disease except :
-bone pain
-osteosarcoma
-delyed healing of fracture***
-pathological bone fracture
122

1124.
916.Keratin formation found in :
-SCC
-ameloblastoma
-adenoid cystic carcinoma
- A and B only ***
1125.
917.Juvenile :
-common
-more sever around incisors and molars ***
-common in male than female
-degenerative disease
1126.
918.Second pharyngeal arch :
-facial nerve
- upper portion of hyoid bone
-A and B ****
- non of the above
1127.
919.Blood Filled blisters on soft palate heals without scaring within 10
days ........ diagnosis :
-traumatic ulcer
-bullosa haemorrhagica *
-hematoma.....
-.....
1128.
920.Best type of flap for palatally impacted canine :
- full thickness ***
-partial thickness
-envelope
1129.
921.Dens Ivaginatus most common in :
-upper central
-upper lateral ***
-upper canine
-equals
1130.
922.Most common 2 malignant tumor of bone respect..:
- osteosarcoma and chondrosarcoma *
- chondrosarcoma and osteosarcoma
-osteosarcoma and ameloblastoma
-.......
123

1131.
923.During rct 1mm shorter file after MAF this called :
- step back ***
-crown down
-.....
-......
1132.
924.Arrange steps befor amalgam :
- caoh ,varnish,base
-caoh,base,varnish***
-base,caoh,varnish
-.......
1133.
925.Primary lymphatic drainage of oral cavity :
- sub mandibular ***
-submental
-..........
-.........
1134.
926.Perforation during rct at function area best ttt :
-MTA ***
-Gic
-caoh
1135.
927.To increase flexibility of the clasp :
- increase length***
- increase taper
-decrease length
-......
1136.
928.best way to ttt sever resorption when planing to do implant at the
upper first molar area is :
-sinus left***
- ridge augmentation
- less successful
-.....
1137. Other questions :
1138. 929.Local anaesthesia content
930.Instrument used to remove fractured tip of currate during scaling
931.Bacteria not found in pericoronitis
124

932.Pt with maler bone defect which syndrome ?


933.2 walls defect best ttt ? Cortical or callen or mixed
934.Multiple head and neck nevi and osteomas ?
935.Optima water fluoridation ?
936.Ttt of trigeminal neuralgia?
1139. 937. Which of the following is the less conservative edge finish
line in perparing crowns
chisel edge
feather edge
chamfer
shoulder***
((I think here we should know the less and the most conservative))
1140. 938. What is the best edge for full metal crown
feather
chamfer***
bevel
chisel
1141. 939.Long story about a young 14 year old patient with excessive
plaque and calculus and u decide to do ultrasonic scaling for him
what is the best ultrasonic to be used
piezoelectric***
ultrasonic
magnetostrictive
sonic
1142. 940.Patient complianing for food accumulation near a molars
area and on x-ray you saw no caries, what is the best way to show the
incipient caries
Periapical x-ray
mirror and explorer
fiber optic transillumination
digital imaging fiber optic transillumination****

125

1143.
1144. 941Long story about patient came after 3 days of trauma he has
lefort1 with displacement and fibrous tissue start to fill the place ..
you will use rowe's forceps (as I remember the name), where you will
put it
Zygomatic process
maxillary tuberosity
hard palate and nasal floor***

126

1145.
1146. 942.A 9 year old child patient when he close his teeth he has C
III occlusion but when he slight open his mouth you see straight face
profile what is the Reason
Face pain
cIII skeletal
functional cIII malocclusion***
127

TMJ problem
((sorry can't remember the option exactly))
1147. 943.Chin cup used for
cIII with man. Prognathism***
cIII with long face
1148. 944.CBC blood test to count?
rbc wbc hb htc - Platelet count***
rbc wbc hb htc - Platelet count cl k
try to make sure from the right answer even if it is not Written
1149. 945.Ankylosis after trauma to primary tooth happen during
formative
eruption
I forget the rest (sorry)
1150. 946.High crystalic enamel has high component of
hydroxyapatite***
calcium apatite

..

128

1151.
1152.
1153.
1154.
1155.
1156.
1157.

947. the space in millimeters that should be between 2 implants


-1
2
3***
4
948. branch of internal carotid

1158. Occipitalo
1159. Ophthalmic artery***1160. facial.

129

1161.
1162.
949._external carotid
Begin
1163. or
1164. divide at hyoid bone.
1165. Sternomastoid cover its course
Medial to internal carotid***

1166. .
950.child has superficial caries at proximal surface of molar what is
your ttt.
1167. Back to back amalgam.
Back to back composite.***
Porcelain crown.
130

1168.
951.6 years child extracted the 2primary molars on both sides the
anteriors not erupted yet your ttt
Removable appliance.***
Band and lope
Distal shoe
1169. Bilateral band and loop thats the right answer if this option
wasnt available then choose removable applaiance
1170.
952._the anathesist told that this operation will take 15minutes. After
cementing the crown ,adjusting occlusion.,washing the field,removal
of rubber dam,what is the first step
1171.
Removal of throat pack***
Write the operaton report
Write post operative instruction.
1172. 953._function of MAF
Determine working length
Determine the final size or shape of apical part.***
For recapitulation.
1173.
954._oxygen flow rate of child(ml/min). The numbers as i remember
nearly:
1-4***
6-9
11_14
1174.
955._the evidence dentistry based on
Reading articles
Study the statistics***
1175.
956.long question and said it is ghost teeth
Dentinogenesis imperficta
Amelogenesis imperficta
1176. Regional odontodysplasia or odontogenesis imperfecta is an
uncommon developmental abnormality of teeth, usually localized to a
certain area of the mouth. The condition is nonhereditary. There is no
predilection for race, but females are more likely to get regional
odontodysplasia. The enamel, dentin, and pulp of teeth are affected,
to the extent that the affected teeth do not develop properly. These
teeth are very brittle. On radiographs the teeth appear more
radiolucent than normal, so they are often described as "ghost teeth
131

1177.
957._child with exophthalmos ,hypoplastic midface diagnosis
Crounz syndrome***
Abert syndrome
1178.
958._stiff impression material
Adition silicon.
Polyether***
Polyvinyl
Alginate
1179. 959.best type of bone resorption in orthodontic
Undermined alveolar
Frontal alveolar***
Hyaline
Inferior
1180.
960.endotreated tooth there is periapical abscess we will retreat it
And want to be sure of removal of all causative organism
1181.
Leave open
Apicectomy and retrograde filling***
Debridement till the radiograghic apex
1182.
961._parakiratinizaton
Not found in gingiva
There is a keratin or keratohyaline granule ***
1183. 962.the technician found blood in impression he will use
Gluteraldyhide***
Acetaldyhide
1184.
963._injection of diazepam there is burning at site of injection due to
Propelyneglycol***
Large needle
1185.
964_synder test measure
Ph of saliva***
Number of microorganisms
To show carious sites
1186. 965.function of barbed broach?
1187. A Retrive vital tissue from very fine canal ***
1188. B.Retrive cotton from the canal enlarge the canal
132

1189. 966.Irreversible pulpitits the pulp is ?


1190. A Vital
1191. B Vital but not normal***
1192. C Dead
1193. 967.the most common failure of Endo ?
1194. A Fracture of instrunment
1195. B Improper obturation***
1196. 968.Post graduate student used Mta as aretrograde , the success
depends on the prevention of ?
1197. A Suture immediately
1198. B Disturbance of wound closure***
1199. 969.Function of post primery ?
1200. A Retain core
1201. B Retain tooth
1202. C used when the remaining tooth structure compromised
1203. D A and C***
1204. 970.Amalgam depends on retention on ?
1205. A Resielence of dentin
1206. B convergence of walls***
1207. C Divergence of walls
1208. 971.Access cavity is?
1209. A Straight
1210. B Divergence***
1211. C convergence
1212. 972.difference between bone and cartilage when increase in size
?
1213. A Apposition
1214. B Hypertrophy
1215. C hyperplasia
1216. D interstitial
1217. 973.when patient first time wear denture what annoy him ?
1218. A Roughness of denture
1219. B Not adaptable with the new vertical dimension***
1220. C Heavy force
1221. 974.Gun shot to mandible and we want a graft best site from?
1222. A Illiac crest superior
1223. B Illiac crest inferior
1224. C Postchondral
1225. 975.We treat graft with cold Hcl to ?
1226. A Kill Hiv and other viruses
1227. B Expose the collagen Fibers***
1228. 976.Trauma to the childs mandible tmj when grow there is pain
and limited mouth opening?
133

1229. Ankylosis
1230. 977.first sign of mandibular fracture
1231. A Bleeding
1232. B Malocclusion***
1233. 978.rapidly growing swelling at angle of mandible pain
parathesia sunshine appearance
1234. A Osteosacrcoma***
1235. B amealoblastoma
1236. 979.primery loss of anterior space affect ?
1237. Aesthetics
1238. Speech
1239. Both***
1240. Anterior space loss
1241. 980.Secretomotor innervation of parotid gland?
1242. A Mandibular N
1243. B Facial .N
1244. C Glossopharyngeal N***
1245. 981.Indirect retainer ?
1246. A prevent the denture from lateral movement
1247. B prevent the denture from moving toward the tissue***
1248. C
1249. D

1250.
1251. 982.you give the pt ID nerve block anesthesia to lower 6 but the
pt still geels pain what is the additional anesthesia you will give him?
1252. A intraligamentary
1253. B infiltration
1254. C Mylohyoid N***
134

1255. 983.when there is no crowding how millimeters is needed in the


jaw to treat 7 mm over jet?
1256. A 10.***
1257. B 12
1258. C 14
1259. D 16
1260. 984. pt.has LA but his skin become warm and he sweating and
fast breathing
1261.
a/hyperthyroidism***
b/hyperglycemia
c/adrenal cisis
1262. 985. diabetic pt. has good oral hygiene and no mobility he need
RPD WHAT Material can we use and can compensate future
resorption
1263. 986.pt. wears complete denture for 5 years continuously he had
very bad oral hygiene and severe inflammation tissues are severly
abused how to manage?
1264. 987. pt. has RPD but has severe inflammation under base
/magor connector and free end saddle when he remove it and use
saline the inflammation relieve but return when he wear denture
1265.
a insufficient relining***
b insufficient stress relief
1266. c.
1267. d.
1268. 988. first event occur in gingivitis
a/increase vascularization***
b/platelet aggregation

135

1269.
1270.
a3
1271.
1272.
1273.

989. minimum duration of taking antibiotic


b4
c5
d6

?To control the child exterimities we use _ .990

.1274

.1275
***Safty belt
Board
after you make MO cavity with composite there is an open .991
contact how can you prevent this open contact
Use packable composite
Use partial strap with wedge
***Use circumfirencial strap with wedge

.1276

you want to make a denture and you found sever anterior .992
?undercut and sever posterior undercut how to manage

.1277

136

.1278
Relief anterior and surgicaly correct posterior
Surgically correct anterior and posterior
Relief posterior and surgically correct anterior

?Minimum crown to root ratio_ .993


2:1
***1:1
1:2

.1279

1280.
keratinized type of pocket we want to change to non keratinized .994
what type of flap
Coronally positioned
Apically positioned
Semilunar flap
diffwrance between periapical absess and peridontal absess _13 .995
Vitality test

137

.1281

.1282

what is the narrowest part of root.996


***Apical constriction
Radiograghic apex

.1283

we can use insted of halogen part of composite light which type .997
of laser
Co2 .1285
Nad yag
***Argon
fracture of nose corrected by .998
Walsham's forceps

.1286

138

.1284

upper .999

.1287
139

?nearly complete,one third of premolar root formed the child is


***9years
8years
11years

.1288

function of resin bonded FPD in support of teeth .1000 .1289


a/discoloration
b/fractured tooth
c\recurrent caries
tooth seldom have two root canal. Seldom means Rarely.1001
a/lower 2
***b/upper 1
if extra canal of lower canine its position .1002
buccal

.1291

140

.1290

***Lingual
Mesial
distal

.1293
.1294

drainage of sub mandibular salivary gland .1003

.1295

The submandibular and upper deep cervical lymph nodes

.1296

which root is more liable to enter sinus when extraction .1004


pt.has lesion on palate related to vital tooth .1005
***a/incisive palatine cyst
b/globulomaxillary cyst
c/aneurysmal bone cyst

141

.1298

.1297

.1292

narrowest part of root canal.1006

.1299

***a/apical constriction
b/radiographic apex

pt.has abscess and fistula after RCTtooth managment .1007


a\retreat
b/extraction
most effetive diagnosis of osseous defect .1008
a/ medical history
***b/ bone sounding x ray
best way to detect working length .1009
**a/x ray
b/instrument with tipped end
c/use calculation

142

.1302

.1301

.1300

1303. 1010.common age of trauma


1304. 2to 3 years
pic bellow 1012 q
1305. 1011.child has cross bite on close but return normal when teeth
apart
1306. 1012.child has injury of anteriors due to trauma no crowding he
is 8 years
a/ class 1
b/class2 division 1***
c/class2 division

2
d/class 3
1307. 1013.q about revascularization what type of anaesethesia used
and type of material above blood clot
1308.
1309. 1014.most inflammatory to pulp
a/zn phosphate***
b/zn polycarboxylate
c/caoh
1310. 1015.pt has high pot.sulphate hypercementosis ant.teeth are
apart bowing legs prominent forehead cotton wool in xray
a/pagets
b/ fibrous dysplasia
1016.test used for moist sterilization
143

1311. 1017.advatage of autoclave over dry oven


kill organisms in less time
1312. 1018.duration needed for removal of non resorbable membrane
1019.10yrs child has dog bite in face managment of wound
a/ligation
b/layered suturing
c/put iodine***
best type of graft .1020

.1313

***a/vascularized
b/non vascularized

.1314
innervation of posterior one third of tongue .1021
The answer here is Glossopharyngial nerve 9 th cranial nerve

144

.1315

.1316

q about 2 years child has white spots on anteriors .1022


managment
nutritional survey

.1317

q about child has periodontitis on molar area what its type .1023

.1318

causative organism of pericoronitis .1024 .1319


It was concluded that the Streptococci milleri group bacteria, wellknown for their ability to cause suppurative infections, are most likely
involved in the pathogenesis of acute severe pericoronitis of the lower
third molar. 2000 American Association of Oral and Maxillofacial
Surgeons
forceps used for excision.1025

.1320

complicated crown fracture affect .1026

145

.1321

a/enamel .1322
b/enamel and dentin
***c/enamel dentin pulp
1323. ENAMEL-DENTIN-PULP FRACTURE (COMPLICATED
1324. DENTAL TRAUMA GUIDE

CROWN FRACTURE)

1325. 1027.sharpest cusp of primary dentition


best material of obturation of primary molars .1028
percentage of canine impaction is more .1029
labial or lingual

.1326

.1327

crowding of primary teeth what indicate for permanent teeth .1030


part of PDL inserted in cementum.1031
infection in diabetes due to .1032

.1329

.1330

*** a neutrophils
B macrophage

.1331

zirconium.1033 .1332
a/high tensile high compressive
***b/low tensile high compressive
c/high tensile low compressive
advatage of custom tray over others .1034
strenggth
rigidity
thickness
146

.1333

.1328

function of postdam of upper denture .1037

.1334

cause of black cast not respond to pickling .1038


over firing

.1335

1336. 1039.removal of diseased root without crown


1337. amputation
1338. Resection
1339. hemisection
1340. 1040.best type of bone under implant
type 2
1341. 1041.best material of implant
titanium
1342. 1042.fracture of coronal part of root of primary tooth
managment
1343. 1043.fracture of middle and apical part of root managment
1344. 1044.insrument used for gingival part of cast
1345. 1045.first method to remove guttapercha
solvent
1346. 1046.pin point exposure and there is hypersalivation
managment
pulpotomy
pulpectomy
direct capping
indirect capping
1347. 1047.mechanism of eruption.
crown and root in same direction or
opposite direction
1348. 1048. about pt has palatal ulcer and fever
147

1349. 1049.mongolism
bisomy21
trisomy21***
1050 dentinogenesis imperfecta affect what
1350. 1051.RL LESION of molar aarea vital teeth no pain
granuloma or radicular cyst
1351. 1052.tooth we cant extract as hyrercementosis
pagets
1352. fibrous dysplasia
1353. AOT
1354. 1053.function of finishline used with all ceramic crown
1355. 1054.matrix band of composite is made of what?
1356. 1055.q about apatient wearing maxillary complete denture with
opposing natural teeth which causes bone resorption which
syndrome?
1357. 1057.all metal crown what type of finish line ?
1358. Chamfer finish line
1359. 1058.after orthodontic treatment patient have some stains ?
1360. Acid pumice microabrasion
1361.
1362.
1363.
1364.

1059.incepient caries detected by ?


Difoti
1060.Dental caries is and Endemic Disease Means
Habitual present in human population

1365. 1061. q about which of the following is concerning a near color


match ?
1366. 1/3
1367. 7/8
1368. Partial veneer
1369. 1062.Cbc refers to
1370. Rbc. Plateles count, HTc,Wb,
1371. 1063.q about a patient that needs and orthognathic surgery in
this case the patiens has a protruded mandible (class III)
1372. Cephalometric X-ray
1373. 1064.innervation of palate and anterior maxillary teeth?
1374. Nasoplatine nerve
148

1375. 1065.treatment of epullisfissuratume?


1376. Allis forceps
1377. 1066 5 years patient with heavy caries index with extracted
lower molar
1378. In x-ray
1379. Distal shoes.***
1380. Stainless steel crown .and loop
band and loop
1381. R.p denture
1382. 1067.composition of enamel organic and inorganic
1383. 1068.fracture in enamel you cant do something I cant remember
what ?
1384. Due to increase organic material
1385. decrease inorganic material
1386. increase in organic material
1387. decrease organic material
1388. 1069.question about plumer vinson syndrome
1389. 1070.we treat grafts with cold Hcl about bone but I cant
remember exactly the question
1390. Kill hiv and any other viruse
1391. Expose collagen fibers
1392. Expose bone .
1393. 1071.Best material for implant ?
1394. Titanuim
1395. 1072.least bone type for implant
1396. Type 4
1397. 1073. best treatment for intruded primery teeth ?
1398. Leave it to re erupt
1399. 1074. diabetic patient cells affected ?
1400. Neutrophil
1401. 1075.patient came to your clinic want to change his old denture
have multi lesion in his mouth and he is wearing his denture always
and he has a bad oral hygene
1402. Recovery treatment
1403. 1076. dens teeth ?
1404. Upper lateral
1405. 1077.child swallow10 mg flouride TTt ?
1406. Milk or any liquid with calcium
1407. 1078.preventive community program?
1408. 1079.local anesthesia content ?
149

1409.
1410.
1411.
1412.
1413.

1080.bacteria not found in newborn


Streptococcus mutans
1081.question about teacher cholins syndrome
Defect of malar bone

1414. 1082. Treatment trigeminal neuralgia?

1415.
150

1416.
1417. 1083. process of bacterial adherence to the tooth surface?
1418. 1089. during maximum mouth opening what happends to both
movement of disc and condyle?
1419. 1090.question about writing a research you will write with
1420. Native language
1421. 1091.znc phosphate cement and Zinc poly carboxylate
contains ?
1422.
1423.
1424.
1425.
1426.
1427.
1428.
and

1092. bleeding of the socket following extraction?


when its always capillary bleeding in nature
to take not less than half day in normal individual
is always favourable when its primery type
can be due presence of nuitreint vessel
answer is 1 ,3 ,4
1093. picture of case of upper second premolar piere abutment
this piere abutment with mobility grade 2

1429. 1094 the most tooth with reccetion and peridontal disease
canine***
premolar
lower fisrt man
151

1430. 1095. After class V GI restoration removal of a thin flush of GI is


done by:
a. Scaller or knife immediately.
b. Finishing stone immediately.
c. Scale or knife next visit
d. Finishing stone next visit
e. a + b.
f. a + d. ***
g. a + c
h. d + c
1431. 1096. Loose enamel rods at the gingival floor of a proximal
amalgam cavity should be removed using :
a. Straight chisel.
b. Hatchet.
c. Gingival curetla.
d. Gingival marginal trimmer. ***
1432. 1097. What can we use under composite restoration for deep
caries
calcium hydroxide***
calcium dihydroxide
zoe
non
1433. 1098 what is the advantages of I- bar ??

esthetic***
easy insertion for patient
undercut on buccal surface
1434. 1099. High mylohyoid crest in patient for complete denture, the
surgeon must avoid vital structure which is:
(during preprosthetic surgery of mylohyoid ridge reduction)
a. Lingual nerve. ***
1435. 1100. Difference between Gracey and universal curette:
a. Section of gracey is hemicircular and in universal triangular.
b. Gracey has one cutting edge while universal has two.
c. Gracey used for cutting in specific area while universal is in any
area.
d. Universal 90 not offset, gracey 60 offset.
e. a and d
f. a, b and c.
g. b, c and d.

152

Minnesota retractor : retract flap and cheek together .1101

.1436



1437. 1102. nicotinic stomatitis

mid palate
1438. 1103. A- HIV :
1439. Elissa test
1440. 1104.maxilla nerve is decending from which foramen ?
foramen rotundum
oramen spinosum
1441. 1105. In hairy tongue, which taste buds affected to i dont
remmber
a. Filli form. ***
b. Fungi form.
c. Foliate.
d. Circumvallates.
.. Pericronitis: Strepto. Malleri or mallery .1106

.1442

1443. 1107. Reiters ( or reiter arthritis or reactive arthritis ) : Tmj


inflammation and ophthalmic disease mostly conjunctivitis

1444. 1108. Papilon le fever : hand and foot keratosis , Periodontitis
affecting both dentitions , early teeth loss , generalized bone
destruction

1445. 1109. Cherbuism : swelling bilateral for
children

1446. 1110. pt with hyperplastic lesion in the floor of the mouth you
want to stain with dye this dye better with normal mucosa than
abnormal but if mixed with toluidine blue give accurate result
a- Toulidim
b- Methyl blue
153

c- Xylo.
d- I can't remember last one
1447. 1111. sharpest cusp for the upper primary molar
mesiolingual***
distilingual
1448. 1112. case about a child comes to your clinic with his parents he
started playing with your instruments and missing around your clinic
the mother said that here child is not good at school nor in life and
she talks to him and he doesnt listen?
1449. Attention deficit hyperactivity disorder (ADHD)
seizure attack .1113

.1450



1451. 1114. what is the dentist between the patient and the dentist?
1452. 1115. origin the lateral ptyrgoid muscle?
1453.
upper head arises from the roof of the intratemporal fossa
(infratemporal surface and the infratemporal crest of the greater wing
of the sphenoid bone )
1454. the lower head arises from the lateral surface of the lateral
pterygoid plate
1455. 1116. a question about the origin of lateral pteregoid from which
bone ?
(infratemporal surface and the infratemporal crest of the greater wing
of the sphenoid bone )

154

1456.

this
155

photo contain the answers of the two previous questions


1457. 1117. Patient after road truama come your clinic with truma to
upper central with no bleeding , after x-ray thier is root fracture with
wide space between two parts, what treatment is recomended ?
1. Extract the entire tooth .
2. Endodontic treatment of coronal part and extract the lower part
surgically.
3. Splint two parts together with fiber splint .***
4. Extraction of coronal part and endodontic treatment to apical part
?What factors affect the success rate of the implant .1118

.1458

Careful patient selection .i


Exacting diagnostic records .ii
Integrated treatment planning .iii
Precise clinical procedures .iv
?How are facial and palatal clefts classified .1119
Class I Cleft lip only

.1460

Class II Cleft lip and cleft palate


Class III Cleft palate only
Class IV Facial cleft

.1459

.1461

.1462

.1463

1464. 1120.-What are the advantages of a continuous suture?

156

1465.
?What are the primary maxillary fascial spaces .1121
,Canine

.1467

.buccal, and infratemporal

.1468

1469. Primary maxillary spaces


a. Canine space
b. Buccal space
c. Infratemporal space
1470. The answer here is all of the above too
?Which bones articulate with the zygoma .1122
Frontal bone .i
Sphenoid bone .ii
Maxillary bone .iii
157

.1471

.1466

Temporal bone .iv


In the human skull, the zygomatic bone (cheekbone, malar bone) is a
paired bone which articulates with the maxilla, the temporal bone,
the sphenoid bone and the frontal bone.i think the answer here is all
of the above
What are the appropriate splinting times for an avulsed tooth, a .1123
?root fracture, and an alveolar fracture
Avulsed tooth 7 days Root fracture 3 months Alveolar fracture 34
weeks
?What risk factors are associated with Latex allergy .1124
Frequent exposure to Latex
History of surgery
Spina bifida

.1473
.1474

.1475

.1

.2

.3

Frequent catheterization

.4

Allergies to certain food, such as bananas, avocados, kiwi fruit, and


chestnuts
?For how long must prophylactic drugs be taken .1125

?What is hemophilia B .1126

.5

.1476

The current recommendation is to take the drugs for 4 weeks

.1477

.1478

Hemophilia B is a congenital bleeding disorder characterized by a


deficiency of clotting
.factor IX

.1472

.1479

.1480

1481. 1127
1482. This is anote book not aquestion which comes in the exam as a
case with picture ..Is benign migratory glossitis (geographic tongue)
associated with any systemic
1483. conditions?
158

1484. Most cases of benign migratory glossitis are associated with


atopy (history of hay
1485. fever, asthma, and eczema) and some human leukocyte antigen
(HLA) types.
1486. Some cases have been associated with fissured tongue and
patients with psoriasis,
1487. especially generalized pustular psoriasis, have a higher
incidence of benign migratory
1488. glossitis

1489.
1490. The picture above is bening migratory glossitis ( geographic
tongue)
1491. Ref , Dental secret
1492. 1128.What is the distance in feets between the dentist and the
nurse while working
1493. A 4ft
1494. B 5ft
1495. C 6ft
1496. 1129.what is the shape of the obturated apex with silver points?
1497. I dunno doctor maybe its this answer after a long time search
1498. It has been observed that in inexperienced hands root canal
instrumentation often results in canals with elliptical or teardrop
shaped foramina. Great attention must be paid to maintaining or
159

developing roundness at the apical foramen if success is to be


achieved with silver cones.
1499. the apical puff is typical, and is looked for as an indication that
the silver cone has been cemented along its entire length
1500. 1130. what are the muscles responsible for smiling?

1501.
1502. 1131.a patient with bad odour what is the gas responsible for
this bad oudor and the dentist could smell it?
1503. This gas packs quite a wallop. Methyl mercaptan is possibly the
most pungent, if not the most common, of all the gases in bad breath
1504. 1132.what is the composition of silver points?
1505. Approx. 99.85 % pure silver; Max. 0.12 - 0.13 % nickel content
1506. 1133.if the patient got paralised in your clinic what is the first
thing to do?
1507. 1134. what is the name of technique used to anesthetise tooth
number 18 (upper right third molar)
1508. Posterior superior alveolar nerve block
1509. 1135.6 y old child vomited on the dentist , what shall the dentist
do?
160

1510.
1511.
1512.
1513.

A shout on him
B kick him out of the clinic
C help the patient peacefully and accept it
D continue working as nothing happens

1514. 1136.
1515. Acase in this picture the patient is 9 years old what is your
diagnosis?
1516. A Trauma
1517. B early loss of teeth
1518. C impacted
1519. 1137.patient comes to your clinic he is sick , he got aposthetic
heart valve , diabetes , hypertension and surgry in his back . he wants
to extract a tooth how to start your management?
1520. 1138.what to mix with acryl to make it hardens faster?
1521. 1139.what is the width of the wax on the patients cast?
1522. 1140.what are the solvents used to remove silver points
1523. 1141.in an orthodontic extrusion to an impacted canine what is
the diameter of the wire ?
1524. I think its 0.012 inches or 0.3 mm
1525. 1142.what is the distance of the opened clift lip ?
1526. 1143.if you have agroup of kids in your clinic and you would like
to advise them how could you divide them?
1527. A 4
1528. B 6
1529. C 8
1530. 1144.Pt. 18 yrs. class II have badly decayed centrals need to be
restore and there minimal overlap, the best restoration:
a. Metal crown
b. Porcelain jacket
c. Veneer
d. Metal ceramic
161

e. Porcelain fused to metal


1531. 1145.Patient with maxillary RPD with missing 16,17,18 and
25,27 and 26 is badly decayed and a non-restorable tooth so the
dentist decided to extract it what is the change in the classification
after extraction:
A. class I
B. class II modification I
C. class II modification II
D. Class III
1532.
1146.Pt. come with severe pain, no response to pulp test when you do
percussion the patient jump, diagnosis is: (No periapical change in
radiograph)
1. Acute apical abscess
2. Chronic apical abscess
3. Symptomatic apical periodontitis.
4. Asymptomatic apical periodontitis.
1533. 1147.Female come with mass on left neck, slow growing before 6
years, the first surgeon said it is a harmless sialodenitis, now CT scan
show mass on submandibular gland, your diagnosis:
A. sialodinitis
B. pleomorphic adenoma
C. adenoid cystic carcinoma.
1534. 1148.Patient come to office with complete denture acrylic in the
upper jaw and partial metallic Class l Kennedy denture in the lower
constructed before one year pt. complain was pain from 15 days in
the lower anterior teeth and around of them what's the suitable
diagnosis:
a. Flexible free area
b. Allergy to acrylic
c. Rough surface in the denture base
d. Sharp end of the flange
1535. 1149.Patient come back after 24 hours of insertion of upper and
low dentures with severe pain in denture bearing area:
A. denture stomatitis
B. overextended flanges
C. suprocclusion
D. denture hyperplasia

162

1536. 1150.. Patient came back trismus:


A. William's technique
B. Gow-gates technique
C. Akinosi technique ***
1537. 1150. Studying the effect af implant diameter and length on
stress distribution which the following correct
1538. 1 diameter has greater effect than do length
2 length has greater effect than do diameter
3 niether diameter not length have effect on stress distribution
4 stress distribution inversly proportional to implant
:The majority of primary herpetic infections are .1151

.1539

.a. Symptomatic
.***b. Asymptomatic. ( not painful )
.c. Proceeded by fever
.d. Accompanied by gingival erythema
***.e. a, c and d
1540. 1152.Primary teeth had trauma, tooth change in color becomes
white yellowish, what should you tell the parents:
a. Pulp is dead.
b. Inflammation of pulp.
c. Calcification of dentin.
d. b & c.
1541. 1153 Patient came to your clinic with dull pain in the #6,no
response to the pulp tester, in radiographs it shows 3mm of
radiolucency at the apex of the root. Diagnosis is:
a) Chronic apical periodontitis.
b) Acute apical periodontitis.
c) Acute periodontitis with abscess.
d) Chronic apical abscess.
1542. 1154.Pt. construct for him a complete denture after few days he
came to u complaining from pain & white spots on the residual ridge
do relief in that area & give him ointment & after few days he came
again complaining the same but in another area the main cause is :
a. Uneven pressure on the crest of alveolar ridge.
163

b. Increase vertical dimension.


1543. 1155.Skeletal bone of skull develops from :
a- Neurocranium ossification.
b- Intramembranous ossification.
c- Endochondral ossification
1544. 1156.Porcelain, highly esthetic, anterior maxilla area, we choose:
OR
Pt. has discoloration on his Max. Ant. central incisors, and u planning
to do veneer for him. Which type of porcelain has high easthetic ?
a. In ceram.
b. Dicor glass reinforced risen.
c. Impress.
1545. 1157.Distal surface for first upper premolar, contact with the
neighboring teeth:
A) in the middle with buccal vastness wider than lingual one.
B) in the middle with lingual vastness wider than buccal one.
1546. 1158.Which condition is an apical lesion that develop acute
exacerbation of chronic apical abscess:
a- Granuloma.
b- Phoenix abscess.
c- Cyst.
d- Non of above.
1159.Periodontally involved root surface must be root planed to:
a. Remove the attached plaque and calculus.
b. Remove the necrotic cementum.
c. Change the root surface to become biocompatible.
d. All of the above.
e. a & b only.
1547. 1160.Female pt. comes with endo treated upper central with M &
D caries and have incisal abrasion. Porcelain veneer is planned with
modification to cover incisal edge. Veneer should end:
a. fourth lingualy 0.5 mm before centric occlusal.
b. fourth 1.5 mm before centric occlusion.
c. fifth 1.5 mm before centric occlusion.
1548. 1161.Old pt. comes with set of compelete denture with tight
denture in morning and become loose later in a day what is the
cause:
164

a. lack of posterior palatal seal.


b. deflecting of occlusion.
c. excessive relining of denture .
d. inelasticity of cheek.
1549. 1162.When the mandible moves to the working side, the
opposite side cusp to cusp contacts in order to balance stresses of
mastication. This relation is called:
1. Working relation.
2. Balancing relation. ( Balancing side ).
3. Occlusal relation.
4. None.
1550. 1163 Teeth selection in setting up is based on these factors:
1. Shade of the teeth.
2. Size and shape of the teeth.
3. Angle of the teeth.
4. 1 and 2.
5. All the above
1551. 1164.q about a patient having some problems in gingiva and
have some inflamations and have leukemia something about blood
was less than?
1552. 1165.it was the same case as in the x-ray below it wasnt that
clear as the picture below ? whats your diagnosis?

165

1553.
1554. A compound odontoma***
1555. B complex odontoma
1556. C soft tissue something
1557. 1166.a child in your clinic which has been anasthetised by
nitruose oxide the child started to laught hysterically without stoping,
what to do ?
1558. a.he wrote some word in English couldnt understand or
remember the world but it might be the right answer
1559. b stop treatment
1560. c wait till the child sleep then continue treatment
1561. d complete treatment
1562. e.
1563. 1167.patient with denture which is formed fromed from acryl
and metal one year ago and the denture was okay and the gingiva
166

too and there is no inflammation and the patient started to complaint


from one month
1564. What is the reason behind that ?
1565. 1168.this questions comes twice in my exam , achild was treated
with a pulpotomy with calcium hydroxide how will you know that the
treatment is going as planned ?
i. patient wont feeling pain
1566. b.the root continued to form normally
1567. c.when he eats he geel some pain
1568. 1169.a picture of a denture which was broken from the frenum to
the posterior the denture was broken twice before and was repaired
what is the reason of breaking it the third time ?
1569. One of the answers was ill fitting denture and thick frenum
1570. 1170. The indication for the use of lingual plate major connector
includes:
1571. For the purpose of retention.
1572. When the lingual frenum is high or when there is a shallow
lingual sulcus.
1573. To prevent the movement of mandibular anterior teeth.
1574. All of the above. ***
1575. Dental decks 641
i. Contraindications for the use of lingual plate major connector :
Crowding of lower anterior teeth .
1576. 1171. The retainer of rubber dam:
OR
In placement of rubber dam:
a. Four points of contact 2 buccally and 2 lingually without rocking. ***
b) 4 points of contact 2 buccally and 2 lingually above the height of
contour.
c) 4 points of contact 2 mesially and 2 distally.
d) 2 points one buccally and one lingually
1577. 1172. Which material the denture base made from it and can
reline later:
1.
Metal
2.
Acrylic***
3.
Combined
167

1578. 1173. Surgery for ridges aims to:


1. Vertical dimension.
3.
Speech.
3. Modify ridge for stability. ***
1579.
1580.
1581.
1582.

1174.space between primery incisors and canine is called


A leeway space
B freeway space
C Primate space ***

1583. N.B
1584. A naturally occurring spacing between the teeth of the primary
dentition. In the maxillary arch, it is located between the lateral
incisors and canines, whereas in the mandibular arch the space is
between the canines and first molars. It also occurs between the
canine and first premolar teeth in adult primates.
1585. 1175. Centric relation is:
a. Teeth to teeth
b. Bone to bone***
c. bone to teeth
1586.Centric relation is the most retruded relationship of the mandible
to maxilla when the condyle in the most posterior unstrained position
in glenoid fossa from which lateral movement can be made at any
given degree of jaw separation.
1587.Centric occlusion the occlusion of teeth when the mandible is in
centric relation to maxilla (maximum teeth intercuspation)
1588.

1176.

1589. We have Tried our best to get the correct answers .the answers
are checked 3 times .if there are some mistakes forgive us it was not
made intentionally .Thanks for this Great Group.

168

169

S-ar putea să vă placă și